Provisional and differential diagnonses

¡Supera tus tareas y exámenes ahora con Quizwiz!

STEM: During a baby check, you notice an infant to have epicanthal folds, a broad nasal bridge, a large tongue, small ears, hypertelorism, a single palmar crease on each hand, and a harsh pansystolic murmur.

- PDx: Downs Syndrome (T21) - DDx: o Patau Syndrome (T13) o Edwards Syndrome (T18) o Foetal alcohol syndrome àThese are the only 3 compatible with life, however 13 and 18 is rare to get to adult hood - Other: o Fetal alcohol syndrome - broad nasal bridges, can have epicanthal folds, can have hypertolism o Turner's can sometimes give some hypertolism but usually have short webbed neck

STEM: The wife of a 48-year-old male patient brings him to the emergency room and says that his memory has progressively gotten worse over the last several years. She also says his personality has been changing. On examination, you note abnormal writhing movements of the man's limbs and hyperreactive reflexes.

- PDx: Huntington's Disease - DDx: o Spinocerebellar atrophy o Space occupying lesion o Frontotemporal dementia o Creutzfeldt-Jakob disease

STEM: A mother brings in her 15-year-old son to see you. His father recently died suddenly due to a heart condition and she would like you to evaluate her son for the same disorder. On exam, the patient is 6 feet tall with long arms and fingers, an inwardly-curving sternum and severe scoliosis.

- PDx: Marfan's Syndrome - DDx: o Klinefelter's (47XXY) o Ehlers-Danlos Syndrome o Fragile X o Loeys-Dietz Syndrome (different genotype but similar phenotype)

A 36-year-old businessman sought medical advice because of a painless mass in the left side of his scrotum, which had been present for three weeks. He was otherwise well apart from a history of surgical treatment for an undescended left testis as a child. Examination revealed a 5 x 3 cm, non-tender, firm mass at the upper pole of the left testis. It was possible to get above the lesion and it did not transilluminate.

Although, testicular cancer is a rare cancer, given this patient is of the right age demographic (20-40 years) and is presenting with a painless non-transilluminating scrotal mass that is definitely scrotal in origin (as implied by the fact that the lesion can "be reached above"), my provisional diagnosis would be testicular cancer specifically a seminomatous germ cell tumour which is the most common type of tumour in this demographic. I can feel quite confident about this provisional diagnosis because · non-tender -> makes inflammatory and infective causes less likely · firmness and failure to transilluminate -> makes a cystic mass less likely o Transillumination involves using a pen torch to shine light from behind a scrotal lump to observe whether the light travels through and illuminates the lesion. The technique helps to assess whether a mass is fluid-filled or not, as fluid will transilluminate and solid masses will not. · ability to get above the lesion -> makes a hernia less likely NB: MY FIRST PRIORITY IN CONSIDERING POSSIBLE DIFFERENTIALS WOULD BE TO IDENTIFY A TORSION Differential diagnoses for scrotal masses o Testicular torsion: medical emergency! Severe pain, <24hours, N/V, high position and transverse lie of testis (Brunzel's sign) and retracted scrotal skin (Ger's sign), may shows discolouration from ischaemia o Indirect inguinal hernia o Infective § Epididymitis: caused by retrograde movement of bacteria from a UTI § Orchitis: inflammation of actual testis, often from mumps § Syphilitic gumma: tertiary syphilis à interstitial inflammation of testes o Cystic § Hydrocele: fluid accumulation within the sac of tunica vaginalis § Epididymal cyst § Haematocele: blood accumulation within sac § Spermatocele: sperm accumulation within sac o Varicocele: bag of worms, abnormal enlargement of the pampiniform plexus o Malignancy: Testicular tumour § Germ cell tumours (90%) · Seminomas · Non-seminomatous germ cell tumours (NSGCT) · Mixed tumours § Sex cord stromal tumours · Leydig cell carcinoma: usually produces androgen, can manifest as precocious puberty, Reinke crystals on histology · Sertoli cell carcinoma: comprised of tubules, clinically silent · Good prognosis, 90% are benign § Mesenchymal neopl

Case: A 34-year-old woman presented with facial rash, marked hair loss, sore joints and previous 4 miscarriages. She has proteinuria and a high ANA titre. Over the last 3 weeks she was fatigued, had recurrent fevers and developed small erythematous papules in periungal area, which were tender but non blanching on palpation.

Anti-phospholipid syndrome on a background of SLE

A 61-year-old woman presented to her GP with a three-week history of cough and chest pain. She had coughed up a small amount of blood-stained sputum. There was no history of fever or night sweats. Examination revealed a thin woman with dyspnoea on mild exertion. There was decreased expansion of the left side of the chest and the percussion note at the left base was stony dull. Auscultation revealed expiratory rhonchi, and there was no friction rub.

Because this woman presents with an episode of small-volume haemoptysis, associated with dyspnoea, asymmetrically decreased chest expansion with ipsilateral stony dullness to percussion and expiratory rhonchi, my provisional diagnosis is pleural effusion secondary to lung cancer. Differential diagnoses would include · respiratory causes o neoplastic § primary § secondary · NB: most lung tumours are secondary lesions rather than primary o infective § bronchitis (+/- coagulopathy) § bronchiectasis § pneumonia § lung abscess § TB o vascular § PE § Vasculitis : Goodpasture's or GPA · cardiac causes o CHF +/- pulmonary oedema o IE (septic embolisms from RH disease)

: A 40-year-old man with a past history of alcoholism presents with intermittent epigastric pain, loose stools and unexplained weight loss. May have presented to the Emergency Department complaining of abdominal pain. Has had recurrent symptoms over 3 years. The pain was of gradual onset over the previous four hours and was now severe and confined to the epigastrium.

Chronic pancreatits Differentials are same as those for acute pancreatitis, but in addition have neoplasm: · neoplastic o pancreatic cancer o oesophageal or gastric cancer - GI o Pancreas: pancreatic cancer o Stomach: perforated PUD, gastritis o Gallbladder: acute cholecystitis, ascending cholangitis o Liver: acute hepatitis o Small and large bowels: IBD, coeliac disease o Kidneys: pyelonephritis, renal calculi - Cardiovascular: o inferior MI, ruptured AAA, aortic dissection and hypovolaemic shock - Respiratory o basal pneumonia and septic shock - Spleen o Infarction Radiation to back à retroperitoneal - Aortic o Aneurysm impending rupture o Dissection (would be acute onset) - Renal o Pyelonephritis o Nephrolithiasis - Life-threatening thoracic pathology o Cardiorespiratory § Inferior MI § Pulmonary embolism

You are an "on-call" surgical resident asked to see a 73 year-old man who died in his sleep 12hrs after undergoing surgery to bypass an occluded right popliteal artery. You have not seen him previously. His wife, who is very keen for you to sign the death certificate, tells you that he was known to have suffered from intermittent claudication in the right leg for the previous six months, with recent onset of nocturnal pain in the right calf. He had received treatment for systemic hypertension for the past 18 years and had a history of angina for the past 2 years.

Differentials for intermittent claudication & nocturnal pain include · vascular - critical stenosis or acute arterial occlusion, DVT (worst case scenario: phlegmasia caerulea dolens) · MSK - arthritis, muscle cramps, chronic compartment syndrome · Neurological - peripheral neuropathy, neurogenic claudication (spinal stenosis) · Other red flag conditions for limb pain o dermatological - cellulitis

24 year old diagnosed with rheumatoid arthritis. At follow-up visits, the patient was noted to have dryness of her eyes, nose, mouth and skin, as well as an enlarged spleen, which was palpable five centimetres below the costal margin, and generalised lymphadenopathy.

Dryness of the eyes and mouth (AKA Sicca syndrome) suggests Sjogren's syndrome, which is an autoimmune disease where lymphocytes infiltrate and destroy the lacrimal and salivary glands. In "secondary" disease, Sjogren's syndrome develops in the context of another connective tissue disease (e.g., RA, systemic sclerosis, SLE, polymyositis). NB: Rarely, Sjogren's syndrome is associated with extra-glandular (pharyngeal, tracheal, vaginal dryness & other similar to scleroderma) complications. 60-80% of patients with Sjogren's syndrome possess anti-Ro/SSA and anti-La/SSB auto-antibodies. Felty's syndrome is a rare complication (<5% of patients) of RA where patient's develop splenomegaly & neutropenia. These patients are at risk of complications due to activation of the reticuloendothelial system and pancytopaenia.

An obese 56-year-old woman with long-standing type 2 diabetes presented to her gynaecologist with PV bleeding, which alarmed her because she had been menopausal for six years. During that time, she had regular Pap smear screening, which had never shown any abnormalities. She had taken hormone replacement therapy (HRT) for several years but had ceased three years ago. There were no other symptoms, and no abnormalities were detected on physical examination. Specifically, the involuted uterus was of normal size, and the cervix was mobile. Provisional, and causes of per-vaginal bleeding

Endometrial cancer ^^ post-menopausal bleeding is considered endometrial cancer until proven otherwise! Only 10% of cases of post-menopausal bleeding will turn out to be cancer but 95% of cases of endometrial cancer will first present with bleeding. NB: post-menopausal bleeding: = per vaginal bleeding more than 1 year after the last menstrual period in a woman who is not receiving HRT or has been on continuous combined HRT for longer than 6months · Structural o Pre-menopausal § Pregnancy-related § Normal uterine or otherwise (ectopic, molar=trophoblastic disease), § ongoing (implantation bleed, inc bleeding with multiples) or otherwise (miscarriage, APH) § Fibroids (leiomyoma) § regress after menopause § Congenital malformations - Bicornuate / septate uterus o Post-menopausal § Genitourinary atrophy** § Polyps (cervical or endometrial)** § Endometrial hyperplasia, adenomyosis § Malignancy^^ - pre-menopausal (cervical, choriocarcinoma), endometrial & vaginal** · Non-structural o Infection: PID (incl cervicitis)/STI or UTI (confused for PV bleed)** o Ovulatory dysfunction: underweight, PCOS o Sexual intercourse or trauma +/- Coagulopathy** o Iatrogenic: HRT withdrawal (poor compliance)**

A 62-year-old woman developed increasing dyspnoea over 5 years. She had suffered from a chronic cough for "many years", and had smoked 40 cigarettes a day. There was a past history of pneumonia 3 years previously and recurrent episodes of "bronchitis" over the past 20 years. Examination revealed a thin woman, dyspnoeic at rest and using her accessory respiratory muscles. She was centrally cyanosed with bilateral expiratory rhonchi on auscultation. There was no evidence of clubbing of the fingers. Her cough was productive of copious mucopurulent sputum.

Given her 5-year history of progressively worsening dyspnoea and chronic cough on a background of extensive smoking history my provisional diagnosis is an infective exacerbation of COPD. Differentials for chronic/progressing dyspnoea · Respiratory o Obstructive lung diseases - COPD, poorly-controlled asthma, bronchiectasis - cystic fibrosis, primary ciliary dyskinesia o Restrictive lung diseases - interstitial lung disease o Vascular - Pulmonary hypertension (unlikely, after 5 years would probably already be dead without treatment) o Neoplastic - primary), secondary (unlikely, after 5 years would probably already be dead) · Cardiovascular - cardiac failure, deconditioning +/- obesity · Neuromuscular - neurodegenerative disease, muscular atrophy NB: most of these pathologies could explain the patient's chronic cough too (+ irritation secondary to GORD)

A 24-year-old woman was referred to the rheumatology clinic for investigation of her arthritis. She was previously well until she developed symmetrical painful swelling of the proximal interphalangeal, metacarpophalangeal, wrist and ankle joints. Examination revealed a thin, pale, ill-looking woman with warm, symmetrical swelling of all involved joints.

Given her age, female gender and the distribution of her arthritis (symmetrical polyarthritis affecting the PIP, MCP, wrist and ankle joints) with associated warmth and swelling, my provisional diagnosis is rheumatoid arthritis. Differential diagnoses for polyarthritis (>5 joints) include · seropositive (connective tissue disease) o SLE o Sjogren's o polymyositis o scleroderma · seronegative (spondyloarthropathies) o psoriatic o IBD-associated · Viral (CAUSING ACTUAL ARTHRITIS, NOT JUST ARTHRALGIAS) o Mosquito-borne § Ross River virus § Barmah Forest virus § Chikungunya o parvovirus B19 o Sexually transmitted or increased risk in IVDU - Hepatitis B or C, HIV Other causes of arthritis that are less likely include possible causes of mono-/oligoarthritis (2-5 joints) · non-inflammatory - o traumatic, o OA (unlikely because of age) · inflammatory o seronegative (spondyloarthropathie) - AS (more common in males & spine), reactive arthritis o crystalline - gout, pseudogout o septic - bacterial, viral, fungal

A 51-year-old woman presented to the Emergency Department with an eight-hour history of severe upper abdominal pain which radiated around to her back and was associated with nausea and vomiting. On examination she was distressed by the pain, with tachycardia, fever, and tenderness in the right upper quadrant (RUQ). She had a past history of episodic epigastric pain, which on one occasion was accompanied by jaundice.

Given her history (upper abdo pain) and examination findings (tender RUQ, fever, tachycardia) and prior history of similar episodes associated with jaundice, my provisional diagnosis is acute cholecystitis, secondary to cholelithiasis. Differentials for epigastric disease · Abdominal pathology o Gallbladder/biliary system: acalculous cholecystitis (5% cholangitis), cholecystitis secondary to CBD obstruction by head of pancreas mass, ascending cholangitis, cholangiocarcinoma o Pancreatitis o Other GI - PUD, SBO o Hepatitis · Retroperitoneal pathology o Renal - pyelonephritis, nephrolithiasis o Vascular - ruptured AAA, aortic dissection · Life-threatening thoracic pathology o cardiac - MI, o pulmonary - pneumonia, pneumothorax

A 27-year-old soldier presented to the Emergency Department with a two-day history of fever, headache and rash. She had recently returned from an army camp where she and two of her companions had developed sore throats but had not received any treatment. Examination revealed an ill-looking woman with haemorrhagic, papular lesions over the forearms and abdomen. Neck stiffness was present and Kernig sign was positive. Fundi were normal. Her vital signs are summarised in the table opposite. HR: 110 BP: 90/60 RR: 24 Temp: 39.4 Urinalysis: pH5.5, protein +ve, blood +ve

Given her history of fever, rash, headache and features suggestive of meningism (nuchal rigidity, positive Kernig's sign) and risk factors including recent time spent in quarter of close confinement with other people, my provisional diagnosis is acute bacterial meningitis which based off her vital signs (tachycardia, hypotension, tachypnoea) has been complicated by shock. This is therefore a medical emergency. Given the presence of a haemorrhagic papular rash and haematuria, this is most likely secondary to a Neisseria Meningitidis (#1) infection. Differential diagnoses include · Infectious o meningitis § other bacterial organisms - #2 streptococcal pneumoniae, haemophilus influenzae · infants: GBS, e coli., listeria (also elderly) § viral - enterovirus (cox-sackie and echo), herpes simplex, lymphocytic choriomeningitis virus, mumps (10% of infections are complicated by meningitis), measles · NB: more common but causes a relatively benign and self-limiting illness § fungal - cryptococcus, coccidioides, candida, aspergillus, histoplasma, § parasitic - toxoplasma, amoeba o parenchymal disease § encephalitis (inflammation of the brain, not just the meninges) § brain abscess o subdural empyema § potential complication of frontal sinusitis, osteomyelitis of the skull vault, or middle ear disease · aseptic meningitis o subarachnoid haemorrhage o malignancies - secondary or primary brain tumours o inflammatory - sarcoidosis, SLE, Behcet's · mimics o peritonsillar & deep neck abscess (could explain neck stiffness & fever)

A 13-year-old boy complained of pain and swelling above his left knee for the preceding three weeks. He remembered injuring the leg whilst playing football and had not paid much attention to the pain until it seemed to be getting worse. He had also noticed a persistent cough for the past week. Examination revealed a hard swelling above the left knee. The remainder of the examination was normal. What do you think is the most likely diagnosis? What else could cause pain and swelling of the leg?

Given his age (10-20 years), gender (male), location (metaphysis of long bone) and his symptoms (progressive pain and cough), my provisional diagnosis would be osteosarcoma with pulmonary metastases. Differential diagnoses · Cysts & Tumours o Benign § cystic lesions - unicameral AKA "simple" bone cyst · common in children/young adults § bone forming - osteoid osteoma § fibrous lesions - endochondroma (cartilage-forming tumour within the shaft), non-ossifying fibroma, osteochondroma (bony outgrowth from the epiphysis of the shaft - has a cartilage cap), o Benign but aggressive § cystic lesions - aneurysmal bone cyst (multiloculated) § osteoblastoma, giant cell tumour (osteoclastoma) o Malignant § primary - · bone forming - osteosarcoma · Ewing's · cartilage forming - chondrosarcoma ·Giant cell - malignant osteoclastoma · multiple myeloma (#1 in adults) § secondary/metastases (usually in adults - often axial) · haematological - lymphoma · solid tumours - prostate, breast, kidney, thyroid, lung Infection o Osteomyelitis · Vascular o Infarct · Trauma o Bone bruising, fracture o soft tissue injury

A 35-year-old woman with a three year history of scleroderma presented with five weeks of increasing dysphagia. This was initially for solids and dry food, but now almost any solid food or even thick soup would cause lower chest discomfort and an inability to swallow. The patient slept on three pillows with the head of the bed raised but still experienced significant reflux each night. Despite a "good" appetite she had lost 8 kg in weight over the last 2 months. On examination the patient appeared thin with tight, thickened skin on her arms, face and upper trunk. Abdominal and rectal examination were normal.

Given that the patient's dysphagia was initially for solids and then progressively also for liquid foods, it is likely that the cause is a mechanical or obstructive pathology rather than a neurological one, so my provisional diagnosis is progressive mechanical dysphagia secondary to scleroderma. Neurological causes of dysphagia present with early difficulty initiating swallowing (liquids or solids) rather than a picture of food 'getting stuck' part way through the process. This might manifest as regurgitation at initiation of swallow or aspiration. Scleroderma causes esophageal dysmotility secondary to smooth muscle atrophy (a mechanical defect). If the patient was taking immunosuppressive treatment for her disease, the dysphagia could also be due to an opportunistic infection (e.g. Candida). Scleroderma can also explain this patient's reflux which arises secondary to reduced LES tone: - Scleroderma patients commonly develop atrophy of the oesophageal smooth muscle, fibrosis of the oesophageal wall and incompetence of the lower oesophageal sphincter, which explains her symptoms of reflux oesophagitis. It is likely that persistent acid reflux into the lower oesophagus has led to progressive scarring and stricture. Carcinoma of the oesophagus and severe reflux oesophagitis are other important differentials which need to be excluded. If the patient was taking immuno-suppressive medication for scleroderma, the possibility of opportunistic infection, (e.g. Candida oesophagitis), should be considered Other causes of dysphagia include · oropharyngeal à NB: more likely to be neurological UMN or LMN lesions depending on the site of lesion Stroke Tumour MND incl. Amyotrophic lateral sclerosis UMN/pseudobulbar palsy autoimmune: MS degenerative: Parkinson's Autonomic dysfunction DM LMN/bulbar palsy autoimmune: Guillan Barre Syndrome NMJ myasthenia gravis oesophageal - NB: more likely to be obstructive or related to motility disorders obstructive intralumenal candidiasis (odynophagia) foreign body intramural oesophageal cancer (adenocarcinoma) stricture (reflux oesophagitis, caustic ingestion) radiation-induced extramural mediastinal tumours/lymphadenopathy retrosternal goitre aortic arch aneurysm paraoesophageal hiatus

Case: A 16-year-old boy was brought by ambulance to ED, after being kicked in the right side of the head during a football match one hour previously. The boy was unconscious but quickly regained consciousness on the field. He became increasingly drowsy, and began vomiting several times. ED saw a tender swelling over the right temporal region. What are your provisional and differential diagnoses?

Given the history of head trauma (especially to the side of the head) and LOC followed by a lucid interval and signs of increased ICP (drowsiness, vomiting), my provisional diagnosis is an extradural haematoma. This is often due to damage to the middle meningeal artery (a branch of the maxillary artery), but could also be due to laceration of the meningeal or diploic (interosseus) veins or venous sinuses).

A 58-year-old man presented to his local GP with a two-hour history of severe chest pain. The pain had commenced while running, and was initially associated with nausea, vomiting and agitation. The patient had experienced similar, less severe chest pain while running over the previous three weeks. He had a 27-year history of type 1 diabetes mellitus. On examination the man was distressed, diaphoretic and mildly obese. Blood pressure was 165/105 mm Hg, pulse rate 114/min with frequent ventricular ectopic beats (VEBs) and there was an S4 heard on auscultation of the precordium. Basal crepitations were audible over both lungs.

Given the history of severe chest pain, associated with nausea/vomiting, diaphoresis, and agitation as well as the patient's history of stable angina, obesity and T2DM, my provisional diagnosis à acute myocardial infarction. Differential diagnoses for acute onset chest pain include (* = Potentially life-threatening emergencies) · cardiac causes: o ischaemic - § demand: tachyarrythmia etc. § supply: ACS (STEMI, NSTEMI, unstable angina), severe aortic stenosis, o infective - pericarditis, o mechanical - cardiac tamponade · vascular: massive PE*, aortic dissection*, ruptured AAA*, o valvular heart disease à aortic stenosis · respiratory causes: o pleural - pneumothorax* o infectious - pneumonia · gastrointestinal causes: o oesophageal - severe GORD, diffuse oesophageal spasm, Boerrhaave's syndrome o other perforated viscus in the region of the epigastrium - PUD, pancreatitis, cholecystitis/cholangitis, · chest wall: o muscular strain, costochondritis, rib fracture (traumatic) · radicular pain: o disc herniation, shingles · other: o psychogenic (anxiety)

Case: A 55-year-old male presented to ED with a 2-hour history of central, retrosternal, crushing chest pain and 10-year background of hypertension and 2 weeks of chest pain on exertion resolving with rest. He is obese with a BMI of 35.0. He is tachycardic, tachypnoeic and hypertensive.

Given the history of severe chest pain, associated with nausea/vomiting, diaphoresis, and agitation as well as the patient's history of stable angina, obesity and T2DM, my provisional diagnosis à acute myocardial infarction. Differential diagnoses for acute onset chest pain include (* = Potentially life-threatening emergencies) · cardiac causes: o ischaemic - § demand: tachyarrythmia etc. § supply: ACS (STEMI, NSTEMI, unstable angina), severe aortic stenosis, o infective - pericarditis, o mechanical - cardiac tamponade · vascular: massive PE*, aortic dissection*, ruptured AAA*, o valvular heart disease à aortic stenosis · respiratory causes: o pleural - pneumothorax* o infectious - pneumonia · gastrointestinal causes: o oesophageal - severe GORD, diffuse oesophageal spasm, Boerrhaave's syndrome o other perforated viscus in the region of the epigastrium - PUD, pancreatitis, cholecystitis/cholangitis, · chest wall: o muscular strain, costochondritis, rib fracture (traumatic) · radicular pain: o disc herniation, shingles · other: o psychogenic (anxiety)

A 65-year-old widow presented to her GP complaining of increasing tiredness and lethargy over the previous 6 months. She had a past history of pernicious anaemia treated with monthly injections of Vitamin B12. On examination she was pale and apathetic. Her face was puffy and there were bilateral xanthelasmata. There was a small, non-tender goitre and evidence of peripheral neuropathy.

Given the patient's age and gender, presenting symptoms (fatigue, lethargy), history of autoimmune disease (pernicious anaemia) and clinical findings (bradycardia, myxedema, facial swelling, peripheral neuropathy), my provisional diagnosis is hypothyroidism secondary to Hashimoto's thyroiditis. NB: Hashimoto's is the most common cause of hypothyroidism in the developed world. Iodine deficiency is the most common cause in the developing world. The causes of hypothyroidism can be classified as follows · primary (95%) - failure of the thyroid to produce sufficient hormone · secondary hypothyroidism due to pituitary disease (e.g. adenoma) and tertiary hypothyroidism due to hypothalamic disease (<5%) Causes of primary hypothyroidism include · autoimmune - Hashimoto's · inflammatory o subacute thyroiditis (de Quervain's - painful, transient) o subacute lymphocytic thyroiditis AKA postpartum (painless) · Iatrogenic - surgical, radiological, pharmacological o surgical o radiotherapy (tx head and neck cancers), radioiodine ablation o antagonist medication · dietary iodine deficiency · neoplastic · infiltrative - amyloidosis, sarcoidosis, Reidel thyroiditis (fibrous - RARE) Other causes of her tiredness and lethargy could include · haematological: anaemia · psychogenic: depression, chronic fatigue syndrome · malignancy Differential diagnoses for a neck lump include · Cyst · Lipoma · Lymphadenopathy (see case protocol 22) o reactive - e.g. infectious: bacterial, viral, parasitic o malignancy - § primary - lymphoma, § secondary - e.g. head/neck cancer (SCC) · Thyroid gland pathology o clinical appearance: midline, rise with swallowing but not tongue protrusion, pause before swallowing o differentials § Goitre (enlarged gland) · euthyroid · hypothyroid · hyperthyroid § neoplasm · benign - adenoma · malignant - papillary, follicular, medullary, anaplastic · Salivary gland pathology o infection o neoplasm (benign or malignant) · Congenital abnormalities o thyroglossal duct cyst o branchial (2nd) cleft cyst (lateral, often appear following a URTI)

An 81-year-old man was brought by ambulance to the Emergency Department with severe back pain. The pain had been intermittent for six days but had become increasingly severe in the past four hours. He had a history of myocardial infarction six years previously and a strong family history of ischaemic heart disease. He was also suffering from hypertension, hypercholesterolaemia and gout. There was no history of gastrointestinal, renal or hepato-biliary disease. Examination revealed a distressed, pale old man with ankle oedema. There was an expansile, pulsating mass 6 cm in diameter, in the epigastrium. A systolic bruit was audible over the mass as well as over the femoral arteries. There was mild cardiomegaly, an S3 gallop rhythm and his chest was clear.

Given the patient's clinical presentation with severe back pain and an expansile abdominal mass on a background of significant personal (prior MI, hypertension, hypercholesterolaemia) and family history of cardiovascular disease, my provisional diagnosis is an AAA. The most common location of an AAA is below the renal arteries and above the aortic bifurcation (L1-L4/5), which has a normal diameter of 2cm. Differentials Important differentials that I would like to consider and rule out in my subsequent investigations include · CVD o MI o Aortic dissection (rare to be distal/abdominal) · renal o nephrolithiasis o pyelonephritis o infarction o tumour · musculoskeletal o vertebral fracture - potentially secondary to an underlying lesion (e.g., osteomyelitis, malignancy) · gastrointestinal o pancreatitis o peptic ulcer o cholecystitis (less likely to present with back pain)

A 13-year-old girl was seen at home by her local doctor. She had been unwell for the past week, with increasing weakness and lethargy, and had also complained of thirst and vague abdominal pains. Her mother noted that the patient had not been eating and had vomited repeatedly over the past two days. Examination revealed a pale, thin, drowsy girl with dry mucosae and ketotic foetor. She was admitted to hospital. HR 118 BP 85/50 RR 18 Temp 37.5 Urine pH7 Protein + Glucose +++ Ketones +++ Blood -ve

Given the patients hypotension and tachycardia, presenting on a background of protracted vomiting, as well as abdominal pain, thirst and lethargy, my provisional diagnosis is hypovolemic shock secondary to diabetic ketoacidosis. Differentials · Diabetic ketoacidosis o First presentation o Poor compliance o Intercurrent illness · Hyperosmolar hyperglycaemic state (unlikely in someone this age) · Other causes of ketoacidosis o Starvation § Esp. if history of eating disorder o Alcoholic (unlikely in someone this age) · Other causes of shock - CHOD, esp. o septic shock with lactic acidosis... primary acute abdomen (could explain the abdominal pain, N&V à then precipitated DKA) § appendicitis, pyelonephritis o hypovolaemic in the context of gastroenteritis, ectopic pregnancy (doesn't explain thirst or vomiting) · Causes of high-anion gap metabolic acidosis other than DKA, lactic acidosis and/or hypovolemia o Uraemia o toxic ingestion (e.g., methanol, ethylene glycol) or medications (paraldehyde, iron, iosoniazid, salicylates, aminoglycosides, theophylline).

A 19-year-old female medical student presented to the Emergency Department with severe bloody diarrhoea. The diarrhoea had commenced three weeks earlier and had gradually worsened, so that each day she was passing 8-10 stools containing stringy mucus and blood. The diarrhoea was associated with crampy lower abdominal pain, urgency and tenesmus. On examination, she was pale, thin, lethargic and dehydrated. There was tenderness in the left lower quadrant without guarding or rigidity, and the bowel sounds were increased. Rectal examination revealed no masses, but blood and mucus were evident on the glove.

Given the protracted (3 week) course of her symptoms, my provisional diagnosis is inflammatory bowel disease. The presence of blood in the stool makes it more likely to be ulcerative colitis. Possible diagnoses would include (most likely causes **bolded**) · inflammatory o Chron's disease · Infectious o bacterial - (invasive or inflammatory diarrhoea): enterohaemorrhagic (OH157:17 shiga-like toxin producing) E.Coli, Shigella, Campylobacter jejuni, Salmonella, Yersinia, C. Difficile (pseudomembranous colitis), o Parasitic - Entamoeba Histolytica · (more common in older people) o Vascular · Diverticulosis · Angiodysplasia · Ischaemic o Neoplastic · CRC

An elderly male patient with history of stage 2 CKD, left ventricular failure, AF, and other comorbidities. 2 days prior had started taking diclofenac for knee pain/osteoarthritis. His other medications include: metoprolol, quinapril, frusemide, digoxin, warfarin. Patient is now presenting acutely with shortness of breath, paroxysmal nocturnal dyspnoea. His BP is 160/90mmHg. What are your provisional and differential diagnoses?

Given this patient clinical presentation with acute dyspnoea, paroxysmal nocturnal dyspnoea on this a background of heart failure, my provisional diagnosis is acute pulmonary oedema from exacerbation of congestive cardiac failure with fluid overload secondary to medication interactions between his NSAID, diuretic and ACEI. à ("triple whammy") on a background of heart failure or kidney failure Other differentials I would like to consider for acute dyspnoea include Cardiovascular - LVF - Acute MI - Valvular heart disease - Arrhythmia - Cardiac tamponade Pulmonary - Pulmonary embolus - Tension pneumothorax - Acute exacerbation of COPD/asthma - Airway obstruction o Pneumonia - Pulmonary hypertension Metabolic - Anaemia - Acidosis Deconditioning - Anxiety, panic attack

A 65-year-old female, who had a 10-year history of hypertension and a myocardial infarction three years previously, presented to the Emergency Department with sudden onset of weakness in the left arm and leg. On examination, her pulse was rapid and irregularly irregular. She had a left VII cranial nerve palsy of the upper motor neuron type and left spastic hemiparesis, with an ipsilateral upgoing plantar response.

Given this patient has had an acute onset of unilateral weakness associated with signs suggestive of an UMN lesion (spastic paresis, CNVII palsy with sparing of the frontalis muscles & a positive Babinski's sign) my provisional diagnosis is stroke - most likely ischaemic because haemorrhagic strokes are more likely to present with the evolution of signs/symptoms over a matter of a few minutes causing gradually increasing focal signs. They are also more likely to present with acute onset headache and with early signs of increased ICP (if large), including vomiting and decreased consciousness. Differential diagnoses I would consider include · vascular - TIA, ischaemic stroke (thromboembolic, hyaline arteriolosclerosis), haemorrhagic bleed (intracerebral, SAH, subdural, epidural), vasculitis · infectious - meningitis, encephalitis, abscess, acute disseminated encephalomyelitis · metabolic o seizure with post-ictal Todd's paresis § temporary weakness or paralysis that occurs after a seizure. Mechanism not completely understood o nonketotic hyperosmolar hyperglycaemic stupor (can present with focal neurological deficits - motor and/or sensory deficits, seizures) · hemiplegic migraine · autoimmune - acute attack of MS · neoplastic - tumour · intoxication

Regina is a 48-year-old with a 20-year history of Type 1 diabetes. You had seen her the previous day and arranged some urgent urine and blood tests. Her main complaint was ankle swelling extending up to her mid-shin, orbital swelling and foamy urine.

Given this patient presents with signs of oedema and frothy urine my provisional diagnosis is nephrotic syndrome secondary to diabetes mellitus. -> Chronic kidney disease

A 23-year-old female presented to her GP with a 4-week history of dry cough following an URTI. On examination, several non-tender enlarged lymph nodes (1.5-2.0cm diameter) were found on the right side of the neck. There was no visible evidence of pharyngitis. FBC and blood film were normal.

Given this patient's age and her history of persistent cervical lymphadenopathy my provisional diagnosis is Hodgkin's lymphoma. Other causes of generalised lymphadenopathy · Neoplastic o Haematological § Non-hodgkin's Lymphoma § leukemia o Diffuse metastatic disease from any primary · Infectious ("reactive") lymphadenopathy o bacterial (TB, syphilis, listeria, brucellosis, cat scratch) o viral (EBV, CMV, HIV) o parasitic (toxoplasmosis) o fungal (histoplasmosis) · inflammatory lymphadenopathy o collagen disease (RA, dermatomyositis, SLE, vasculitis, Sjogren's) o infiltrative disease (sarcoidosis, amyloidosis) · iatrogenic o drug hypersensitivity - anticonvulsants (including phenytoin), allopurinol, iosoniazid o serum sickness · Other o specific storage disorders - Gaucher, Neimann-Pick NB: many of these conditions (e.g. drug hypersensitivity, serum sickness) do not explain the patient's weight loss. Causes of localised lymphadenopathy (anatomical) · cervical - o head & neck malignancies: oral cavity, pharyngeal, laryngeal carcinomas (usually SCC's), thyroid cancers, skin cancers o infections (pharyngitis, dental infections, salivary gland infections, conjunctivitis) o (paediatrics) Kawasaki's · supraclavicular o right - mediastinal, bronchogenic, oesophageal cancers o left - gastric, CRC, pancreatic, renal, testicular/ovarian cancers + as per right side · axillary o breast cancer o cat scratch fever · epitrochlear (ALWAYS PATHOLOGICAL) · inguinal o leg wound/ulcer infection, osteomyelitis o STI's - gonorrhoea, chlamydia, syphilis, HSV o pelvic malignancy (cervix, vulval, ano-rectal) · In the distribution of a draining wound o outdoors enthusiast - skin cancer It is possible to vaguely differentiate between possible causes of lymphadenopathy based on whether it is painful or not: · painful - infectious o EXCEPT TB - typically painless · not painful - malignant, inflammatory, infiltrative Causes of night sweats include · malignancies o haematological (lymphomas or leukemias) o solid tumours (prostate, renal cell, germ cell) · infections o HIV, TB o Bacterial - endocarditis, osteomyelitis, pyogenic abscess o parasitic - malaria · endocrine o sex-hormone related § vasomotor symptoms of menopause § drugs (e.

A 36-year-old woman presented to her local doctor after noticing a lump in her breast. Her sister had recently had a benign breast lump removed. Examination revealed a 1 cm mass in the lower outer quadrant of the right breast. The lesion was firm and non-tender, without attachment to deeper structures. There was no axillary lymphadenopathy.

Given this patient's age and the fact that the lump is firm but not stony hard, not tethered and there is no apparent lymphadenopathy, my provisional diagnosis is that this is a benign fibrous or cystic lesion. Provisional: fibrocystic changes Causes of palpable breast masses include · Cystic/Endocrine o fibrocystic change (endocrine sensitive metaplasia) § presents as bilateral "lumpy-bumpy" breasts, with symptoms (tenderness, enlargement) that may fluctuate with the menstrual cycle · typed as non-proliferative and proliferative o galactocele (pregnant/breastfeeding women) · infectious o mastitis +/- abscess (pregnant/breastfeeding women) § NB: inflammatory cancers may be mistaken for this! · fat necrosis (usually develops secondary to trauma (incl. biopsy), radiation) · Neoplastic o Benign tumours § fibroadenoma (breast mouse) § Phyllode's tumour · stromal, similar to fibroadenoma § Intraductal papilloma § lipoma o Malignant § carcinoma · carcinoma in situ · invasive ductal (70-80%) · invasive lobular o presents as prominent diffuse thickening rather than as a mass · inflammatory § other - angiosarcoma

A 27-year-old woman presented to her GP with a persistent facial rash and loss of scalp hair. She gave a history of Raynaud phenomena occurring during the previous three winters, and glomerulonephritis several years ago. Examination revealed an erythematous rash over the malar region of her face and bridge of the nose. Her hands were pale and cold, with soft tissue swelling of the metacarpophalangeal joints of both hands.

Given this patient's age, gender (SLE most commonly affects women of childbearing age) and presentation with a malar rash, small joint polyarthritis, alopecia, Rayneud's phenomenon & past history of glomerulonephritis, my provisional diagnosis is systemic lupus erythematous, likely complicated by nephritis as indicated by the positive urine analysis and hypertension. Differentials I'd like to consider and attempt to rule in/out · drug-induced lupus · other autoimmune conditions o connective tissue disorders - scleroderma, RA, Sjogrens, adult Still's disease, mixed connective tissue disease o myopathies - polymyositis, dermatomyositis o seronegative spondylarthropathies - psoriatic arthritic o Vasculitis An alternative diagnosis specifically for her facial rash, includes rosacea.

A 45 year old woman presented to her local doctor with a three week history of easy bruising and menorrhagia, associated with increasing fatigue and intermittent fever. Physical examination revealed a pale, thin person with prominent ecchymoses on her arms, legs and abdomen. The spleen was "tippable" beneath the costal margin and there was slight bony tenderness over the sternum.

Given this patient's features of anaemia (fatigue, pallor), thrombocytopaenia (easy bruising & menorrhagia), leukopenia/immature leukocytes (intermittent fever), bony tenderness and splenomegaly my provisional diagnosis is leukemia. Given the acute course of her symptoms, I suspect this is an acute leukemia, and given her age (45 years old), splenomegaly & acuity of the illness most likely acute myeloid leukemia. Differential diagnoses for pancytopaenia I would like to consider include: · other haematological malignancies - o other myeloproliferative conditions § ALL (more common in children) § blast crisis in CML § (CLL is usually very indolent and diagnosed in older patients) o myelodysplastic disorders o bone marrow infiltration secondary to lymphoma (à could also explain splenomegaly) o multiple myeloma (bony tenderness) · causes of bone marrow failure o aplastic anaemia o Evan's syndrome (see below) o drug-induced (cytotoxic) o nutritional § folate or vitamin B12 deficiency § excess alcohol intake · hypersplenism (could explain fever, anaemia, splenomegaly) à CHINA o congestive: cirrhosis, portal/splenic vein thrombosis o haematological: haemoglobinopathies, enzymopathies, spherocytosis o infection (see below) o neoplastic (as above) o autoimmune and infiltrative: SLE, RA (Felty's syndrome), sarcoidosis, amyloidosis · infections (many of these have a tropism for RBCs, the liver, and potentially bone marrow) o viral - § EBV, CMV, § HIV § hepatoviruses o protozoa - Malaria

A 63-year-old woman was referred to her local hospital for assessment of ankle oedema and weakness. She had been divorced for 23 years and lived alone. She had previously worked as a cook in the local RSL club but had been on sickness benefits for the past 15 years because of depression. She smoked 40 cigarettes a day and drank 5-6 glasses of sherry per day. Examination revealed a woman who looked older than her stated age. Her gait was ataxic. Pitting oedema was present in both lower limbs. The abdomen was distended and tense, with shifting dullness. The liver was not palpable; however, the spleen was palpable 3 cm below the costal margin. Rectal examination revealed bleeding haemorrhoids.

Given this patient's findings of ascites, bilateral lower limb pitting oedema, splenomegaly associated with an ataxic gait and a chronic history of excessive alcohol consumption, my provisional diagnosis is cirrhosis of the liver secondary to chronic alcohol abuse complicated by portal hypertension and hypoalbuminemia. Important differentials I would like to consider include · Other causes of cirrhosis · chronic viral hepatitis · NASH · inherited - Haemochromatosis, Wilson's, a-1 antitrypsin, porphyrias, glycogen storage disorders (paediatric) · autoimmune - autoimmune hepatitis, PSC, PBC · Other causes of ascites (based on the SAAG, see later q) · Portal hypertension/Transudative § thrombosis - pre-hepatic: portal vein, post-hepatic: Budd Chiari (hepatic v.) § Right heart failure ("congestive hepatopathy") · Deranged oncotic gradient/exudative § Intravascular hypoalbuminemia - nephrotic syndrome § Peritoneal maliganancy/infection etc

A 25-year-old woman presented with anorexia, lethargy and pallor. She had a long history of recurrent urinary tract infections, and had suffered from several episodes of acute pyelonephritis during her childhood, though none had occurred in the last five years. Over the past 12 months she had noticed increasing nocturia.

Given this patient's history of anorexia, lethargy, pallor and increasing nocturia, raised urea, creatinine and low bicarbonate, my provisional diagnosis is chronic kidney disease. The most common causes of CKD in adults in Australia are DM and HTN. But given this patient's age I would be more concerned about the glomerulonephritis or a possible congenital abnormality predisposing to renal failure. Given this patient's long history of UTI's, repeated episodes of acute pyelonephritis I am concerned that she has develop CKD secondary to reflux nephropathy. NB: nocturia and hypertension are classic early signs of CKD and reflux nephropathy accounts for 15-20% of CKD in children and adolescents. Causes of chronic kidney disease can be classified as glomerular, tubulointerstitial, vascular and post-renal obstructive · vascular o large/medium vessel: bilateral renal artery stenosis (atherosclerosis) à ischaemic nephropathy o small vessel: hypertensive nephrosclerosis (#2 most common) o sickle cell disease · glomerular o Primary and secondary causes of glomerulonephritis (#3 most common) manifesting with nephritic or nephrotic syndromes o Other § Diabetes mellitus (#1 most common) · tubulointerstitial o drug-induced § drugs causing acute interstitial nephritis (AIN): antibiotics, NSAIDs, 5-ASA, PPIs § other: · analgesics (paracetamol, formerly phenacetin) · cisplatin, ciclosporin/tacrolimus, · lithium toxicity o systemic disease - autoimmune diseases o toxin-induced - heavy metal · congenital o dysplasia/hypoplasia/agenesis o polycystic kidney disease o Reflux nephropathy o posterior urethral valves · post-renal obstructive o malignant § multiple myeloma § bladder, invasive prostate or cervical cancer o severe BPH

Case: A 35-year-old woman was seen by her local doctor because of increasing dyspnoea over the previous 12 hours. She had suffered from asthma since childhood, requiring numerous hospital admissions, and consequently was reluctant to consult doctors or to go to hospital. Her only medication was a salbutamol puffer, used as required. She had been suffering from an upper respiratory tract infection for the past three days and had a cough productive of yellow sputum. On examination, the woman was dyspnoeic at rest, having difficulty in carrying out a conversation. She was pale but not cyanosed, the chest was hyperinflated with prominent tracheal tug, and she was using accessory muscles of respiration. Bilateral inspiratory and expiratory wheezes were heard on auscultation. She was commenced on antibiotics, nebulised salbutamol and prednisone tablets, but refused admission to hospital.

Given this patient's history of asthma and recent URTI, and the fact that she is presenting with signs and symptoms of an acute airway obstruction (tracheal tug, use of accessory muscles, audible wheezes), my provision diagnosis is status asthmaticus. This is a medical emergency and needs to be managed in a high-acuity clinical setting (e.g., resuscitation bay or ICU). Definition: "status asthmaticus" refers to an acute exacerbation of asthma that is unresponsive to bronchodilators The clinical presentation of asthma is episodic bouts of dyspnoea, wheeze, chest tightness and cough that is often worse at night-time/early morning. NB: it is not uncommon for adults to only have cough and no other symptoms in mild cases. Differentials: - Anaphylaxis - Foreign body obstruction - Pneumothorax - PE - Pneumonia - Bronchitis

A 48-year-old man presented to his GP with a two-week history of sore throat, swollen lymph nodes in the neck and 7 kg weight loss. He had also noted drenching night sweats during the last week. Examination revealed pharyngitis with white plaques on the soft palate and tonsillar area, and swelling of the lymphoid tissues of the oropharynx. Several lymph nodes of 2-3 cm diameter were readily palpable in the submandibular region, while nodes of a similar size were present in the left axilla and even larger nodes in the groin of up to 4cm

Given this patient's history of generalised lymphadenopathy associated with constitutional 'B' symptoms (night sweats, weight loss), my provisional diagnosis is lymphoma, most-likely non-Hodgkin's lymphoma which is more common than Hodgkin's lymphoma in a patient of this age. My differential diagnoses include Other causes of generalised lymphadenopathy · Neoplastic o Haematological § Hodgkin's Lymphoma § leukemia o Diffuse metastatic disease from any primary · Infectious ("reactive") lymphadenopathy o bacterial (TB, syphilis, listeria, brucellosis, cat scratch) o viral (EBV, CMV, HIV) o parasitic (toxoplasmosis) o fungal (histoplasmosis) · inflammatory lymphadenopathy o collagen disease (RA, dermatomyositis, SLE, vasculitis, Sjogren's) o infiltrative disease (sarcoidosis, amyloidosis) · iatrogenic o drug hypersensitivity - anticonvulsants (including phenytoin), allopurinol, iosoniazid o serum sickness · Other o specific storage disorders - Gaucher, Neimann-Pick NB: many of these conditions (e.g. drug hypersensitivity, serum sickness) do not explain the patient's weight loss. Causes of localised lymphadenopathy (anatomical) · cervical - o head & neck malignancies: oral cavity, pharyngeal, laryngeal carcinomas (usually SCC's), thyroid cancers, skin cancers o infections (pharyngitis, dental infections, salivary gland infections, conjunctivitis) o (paediatrics) Kawasaki's · supraclavicular o right - mediastinal, bronchogenic, oesophageal cancers o left - gastric, CRC, pancreatic, renal, testicular/ovarian cancers + as per right side · axillary o breast cancer o cat scratch fever · epitrochlear (ALWAYS PATHOLOGICAL) · inguinal o leg wound/ulcer infection, osteomyelitis o STI's - gonorrhoea, chlamydia, syphilis, HSV o pelvic malignancy (cervix, vulval, ano-rectal) · In the distribution of a draining wound o outdoors enthusiast - skin cancer It is possible to vaguely differentiate between possible causes of lymphadenopathy based on whether it is painful or not: · painful - infectious o EXCEPT TB - typically painless · not painful - malignant, inflammatory, infiltrative Causes of night sweats include · malignancies o haematological (lymphomas or leukemias) o solid tumours (prostate, renal cell, germ cell) · inf

A 49-year-old male academic was admitted for investigation of vomiting and diarrhoea for the past 24 hours. In the last 3 hours he had noted that his motions had been of a black and tarry nature. He had a past history of intermittent epigastric pain. On examination the patient appeared pale and sweaty, with tenderness in the epigastrium. Rectal examination revealed black liquid faeces.

Given this patient's history of intermittent epigastric pain and melaena, my provisional diagnosis is hypovolemic shock secondary a massive upper GI bleed, likely from a peptic ulcer that has eroded into a nearby vessel. This is a medical emergency. My first priority before formulating a diagnosis for this patient is to admit him to a resuscitation bay so that he can be appropriately managed whilst hemodynamically unstable. Melaena refers to black, tarry, foul swelling stool. Melaena represents blood that originated from the upper GI tract that has spent sufficient time in transit for the Hb to be chemically altered. The "upper" GI (UGI) tract (AKA foregut) is roughly defined as the portion of the tract that is proximal to the Ligament of Trietz AKA the suspensory ligament of the duodenum. Beyond this ligament is the midgut and then the hindgut, which begins 2/3 the way along the length of the transverse colon. Haematochezia refers to the passage of bright red stools per rectum. Usually, it represents blood that originates from the midgut or hindgut; however it could theoretically arise from and UGI bleed if it is massive and rapid enough. The most common causes of upper GI bleeds (causing haematemesis & melaena) include ulcers (PUD: duodenal > gastric), erosions >> 85% Oesophageal varices >> 5% Mallory-Weiss tear 5% other tumours - esophageal, gastric (both adenocarcinomas) vascular lesions (congenital - AVMs e.g. Dieulafoy's disease, acquired - angiodysplasia) aortic enteric fistula (unlikely if the patient doesn't have an aortic stent) Mimics Haematemesis - epistaxis, haemoptysis Melaena - benign causes - iron supplementation, liquorice-ingestion, activated charcoal NB: cannot attribute cause of bleeding to coagulopathy, but is important to identify because it may account for severity of bleeding For differentials of epigastric pain - GI o Pancreas: pancreatic cancer o Stomach: perforated PUD, gastritis o Gallbladder: acute cholecystitis, ascending cholangitis o Liver: acute hepatitis o Small and large bowels: IBD, coeliac disease o Kidneys: pyelonephritis, renal calculi - Cardiovascular: o inferior MI, ruptured AAA, aortic dissection and hypovolaemic shock - Respiratory o basal pneumonia and septic sho

A 49yo male was admitted for investigation of vomiting and diarrhoea for the past 24hrs. In the last 3hrs, he noticed his motions had been of a black, tarry nature. He had a past hx of intermittent epigastric pain. On exam, the patient appeared pale and sweaty, with tenderness in the epigastrium. Rectal examination revealed black liquid faeces.

Given this patient's history of intermittent epigastric pain and melaena, my provisional diagnosis is hypovolemic shock secondary a massive upper GI bleed, likely from a peptic ulcer that has eroded into a nearby vessel. This is a medical emergency. My first priority before formulating a diagnosis for this patient is to admit him to a resuscitation bay so that he can be appropriately managed whilst hemodynamically unstable. Melaena refers to black, tarry, foul swelling stool. Melaena represents blood that originated from the upper GI tract that has spent sufficient time in transit for the Hb to be chemically altered. The "upper" GI (UGI) tract (AKA foregut) is roughly defined as the portion of the tract that is proximal to the Ligament of Trietz AKA the suspensory ligament of the duodenum. Beyond this ligament is the midgut and then the hindgut, which begins 2/3 the way along the length of the transverse colon. Haematochezia refers to the passage of bright red stools per rectum. Usually, it represents blood that originates from the midgut or hindgut; however it could theoretically arise from and UGI bleed if it is massive and rapid enough. The most common causes of upper GI bleeds (causing haematemesis & melaena) include ulcers (PUD: duodenal > gastric), erosions >> 85% Oesophageal varices >> 5% Mallory-Weiss tear 5% other tumours - esophageal, gastric (both adenocarcinomas) vascular lesions (congenital - AVMs e.g. Dieulafoy's disease, acquired - angiodysplasia) aortic enteric fistula (unlikely if the patient doesn't have an aortic stent) Mimics Haematemesis - epistaxis, haemoptysis Melaena - benign causes - iron supplementation, liquorice-ingestion, activated charcoal NB: cannot attribute cause of bleeding to coagulopathy, but is important to identify because it may account for severity of bleeding For differentials of epigastric pain - GI o Pancreas: pancreatic cancer o Stomach: perforated PUD, gastritis o Gallbladder: acute cholecystitis, ascending cholangitis o Liver: acute hepatitis o Small and large bowels: IBD, coeliac disease o Kidneys: pyelonephritis, renal calculi - Cardiovascular: o inferior MI, ruptured AAA, aortic dissection and hypovolaemic shock - Respiratory o basal pneumonia and septic sho

An 18-year-old woman was brought by ambulance to the Emergency Department with uncontrolled haemorrhage following extraction of a wisdom tooth at a dentist's surgery. She had always "bruised easily" and often experienced "heavy periods", but there had been no previous operations. On examination, she was pale and extremely anxious, with cool extremities. Vital signs were as shown opposite: HR 110 BP: 90/60 RR: 18 Temp: 36.2

Given this patient's relatively young age and history of lifelong 'easy bruising' and 'heavy periods' she is likely suffering from previously undiagnosed bleeding disorder. Causes of bleeding disorders can be classified as disorders of primary or secondary haemostasis. Disorders of primary haemostasis are due to an inability to form an adequate platelet plug. Disorders of secondary haemostasis are due to an inability to form an adequate fibrin clot. Given the pattern of bleeding (mucocutaneous rather than deep tissue haematoma formation) I suspect she is suffering from a disorder of primary haemostasis. The most common congenital bleeding disorder is von Willebrand Disease which results in abnormalities in primary and secondary haemostasis. Provisional: Von Willebrand disease - Easy bruising, menorrhagia and haemorrhage after dental procedures suggest a disorder of primary haemostasis Differentials: Disorders of primary haemostasis · Platelet disorders Thrombocytopaenia -acquired - ITP, HUS Platelet function abnormality - congenital - Bernard-Soulier syndrome(a severe bleeding disease due to a defect of GPIb/IX/V, a platelet complex that binds the von Willebrand factor) - acquired - uraemia, aspirin-use Disorders of vessel wall - congenital - hereditary haemorrhagic telangiectasia, Ehlers-Danlos syndrome(group of genetic conditions that affects connective tissues, mainly skin/joints/walls of blood vessels), -acquired - scurvy, vasculitis Disorders of secondary haemostasis Clotting factors/co-factors -Haemophilia A or B(genetic mutation causes deficiency of coagulation factors (VIII or IX) - vitamin K deficiency)\ Excessive fibrinolysis - congenital - alpha2-antiplasmin and plasminogen-activator inhibitor 1 deficiencies - acquired - snake envenomation Sometimes bleeding disorders are caused by condition that impair primary and secondary haemostasis - von Willebrand Disease ·-liver failure (decreased synthesis of thrombopoetin and also clotting factors and fibrinogen.) -DIC - systemic activation of coagulation pathways from endothelial injury leading to massive consumption of platelets, coagulation factors and fibrinogen.

A 36-year-old man presented to the Emergency Department with severe right loin pain. The pain had been present "on and off" for the past six hours, with each attack of pain lasting approximately 30 minutes. The patient recalled a similar episode of pain six months previously that had resolved spontaneously. Examination revealed a distressed man, pale and tachycardic. The abdomen was soft and non-tender, and no other abnormalities were detected on physical examination.

Given this patient's severe episodic 'colicky' loin pain with a urine sample that is positive for proteins and blood and an abdomen that is soft & non-tender, my provisional diagnosis is renal colic secondary to a renal or ureteric calculi. The patient's clinical features are due to spasmodic contraction of a distended ureter that is partially or completely obstructed. In a patient presenting with the typical "loin to groin" pain of renal colic differentials I would also consider, include alternative · causes of ureteric obstruction o intra-luminal: calculi, blood clots (PKD) o intra-mural: tumour (TCC), strictures (iatrogenic, chronic calculi) o extra-mural: tumour (RCC), endometriosis, retroperitoneal fibrosis · other life-threatening causes of pain (NB: most would present with constant pain rather than colicky) o renal pathology § infection: pyelonephritis § vascular: infarction, venous thrombosis o retroperitoneal pathology § aortic (aneurysm, dissection) o abdominal pathology § biliary colic (right-sided), § perforated intra-peritoneal viscus - peptic ulcer, appendicitis (right-sided), pancreatitis · NB: patients are likely to be very still o pelvic pathology § (female) gynaecological - (female) ectopic pregnancy, ovarian torsion § (males) causes of acute scrotal pain - testicular torsion · MSK - muscular strain · Neurological - radiculitis due to herpes zoster infection or nerve root compression

A 30-year-old lady presents to ED at 3 weeks postpartum with sudden onset of pleuritic chest pain, shortness of breath and tachycardia (HR 122). Her BP is 100/60, RR 30 and temperature 37.8ºC

Given this patient's sudden onset of pleuritic chest pain and the presence of clinical risk factors for DVT including pregnancy/post-partum period (& possible recent abdominal surgery -> caesarean section), my provisional diagnosis is pulmonary embolism. (Not pneumothorax because that is within days) Despite the fact that this patient's blood pressure is technically normal, it is on the low-end and I am worried that this patient may have had a sub-massive PE that could be causing/cause obstructive shock. Hypotension is a late sign in shock and as such I am concerned that this patient could decompensate. This is therefore a medical emergency. - Classic triad of PE: acute onset of pleuritic chest pain, dyspnoea and haemoptysis +/- fever - Risk factors: pregnancy/postpartum, immobility, Caesarean section - Tachycardia, tachypnoea = compensatory mechanisms indicating V/Q mismatch -> due to thromboembolic occlusion of a lobar or segmental pulmonary artery Differential diagnoses I would like to consider, include · Acutely life-threatening causes of pleuritic chest pain · Respiratory - airways (asthma exacerbation), parenchymal (pneumonia, acute pulmonary oedema), pleural (PTX, effusion), Spontaneous tension pneumothorax · Cardiovascular - MI, aortic dissection · Musculoskeletal · Psychogenic · Other

70yo man with retrosternal tearing chest pain radiating to back. Background of HTN. On exam, BP is 200/100mmHg, weak radial pulse, collapsing carotid pulse, and diastolic murmur. eGFR is 23 and creatinine is high (~240). On urinalysis, there is proteinuria + and haematuria +++. Troponins are normal.

Given this patients age, gender (male), history of HTN and clinical presentation, my provisional diagnosis is aortic dissection. NB: collapsing pulse and diastolic murmur is due to acquired aortic regurgitation due to proximal extension of the dissection Differential diagnoses I would like to consider include other life-threatening ... conditions As well as further ... conditions · Cardiovascular o ACS o ruptured AAA o Pericarditis, Myocarditis · Respiratory o PE o PTX o pneumonia · GI o Boerhaave syndrome o Pancreatitis o Cholecystitis o PUD

46 year old man presenting with a coarse, scratchy voice, pain with swallowing and difficulty swallowing solids. The following is an image seen on direct laryngoscopy.

Given this patients age, gender, smoking history and dysphonia my provisional diagnosis is laryngeal cancer à 95% of which are squamous cell. · >3 weeks = chronic à laryngeal cancer until proven otherwise · NB: 1/3 of patients present due to respiratory infection or widespread metastases Differential diagnosis: · benign vocal fold lesions o polyps (most common): develop due to irritation of the vocal fold (see image to right) o nodules (fibrotic): can be treated conservatively or via steroid injection or surgical resection § NB: the aetiology of benign vocal fold polyps and nodules are very similar. o cysts (paediatric): caused by obstruction of mucus secreting glands o granuloma § NB: formation may be triggered by endotracheal intubation. o Reinke oedema · infiltrative disease of the larynx - amyloidosis, sarcoidosis, GPA · premalignant lesions o leukoplakia o papillomatosis (HPV-associated warts) o pachyderma (rough/thickened mucosa on the vocal folds) · other causes of hoarseness o neurological (UMN: stroke or MND, LMN: recurrent laryngeal nerve damage, NMJ: MG) o acute § voice misuse § viral laryngitis §

A patient with chronic Crohn's disease has been taking azathioprine for years. He has a low mood and starts taking sertraline. Develops an acute swollen toe. Provisional

Gout

An 81-year-old woman was brought to the Emergency Department after a fall whilst shopping. She had severe pain in her left hip and was unable to stand. Her left leg was shortened and externally rotated. Radiological examination confirmed the clinical diagnosis of a fractured neck of the femur, and appropriate orthopaedic management was instituted. Two days after admission to the hospital, she developed a low-grade fever (38.2 oC) and at this stage examination of the chest revealed bilateral basal crackles and occasional coarse wheezes. A day later she developed a cough productive of mucopurulent sputum and her temperature rose to 39.4 oC. Appropriate investigations were ordered and presumptive treatment commenced. However, at 1.00 am the following morning she became acutely dyspnoeic, collapsed and could not be resuscitated. An autopsy was performed.

My provisional diagnosis is a right proximal femur/neck of femur fracture (as right leg was shortened and externally rotated) secondary to osteoporosis given the minimal trauma history (fall from standing height) and risk factors of age (post-menopausal), female and history of previous minimal trauma fracture. My differential diagnoses of secondary osteoporosis and therefore fracture includes: · Metabolic o Osteomalacia o Renal osteodystrophy o Paget's disease o Hyperparathyroidism o Hyper/hypothyroidism o Cushing's Disease · Neoplastic o Primary: § Osteosarcoma: 13yo boys § Multiple myeloma § Leukaemia § Lymphoma o Secondary: § Breast cancer + bony metastases § Colorectal cancer + bony metastases · Infective o Osteomyelitis · Drug-induced o Steroid use o Anticonvulsants · Nutrition - Vitamin D or calcium deficiencies · Trauma: fall could be greater or intentional Given her age (>50 years), the most likely disease contributing to this fragility fracture (fall from standing height) is osteoporosis. Osteoporosis is a disease where there is decreased bone mass and quality. In a woman of this age, other diseases that may generally weaken bone or predispose to fragility fractures, include · Generalised weakening of bone o endocrine - primary hyperPTH, Cushing's, hypogonadism, T1DM, hyperthyroidism o metabolic - renal osteodystrophy, osteomalacia (calcium and/or vitamin D deficiency), Paget's o malnutrition - e.g., eating disorders, malabsorptive conditions (e.g., coeliac) o alcohol & smoking o drugs § corticosteroids, anti-estrogens and anti-androgens, § anticonvulsants (induce cyp450 enzymes which inactivate vitD), PPIs (decrease absorption of Ca2+ from the stomach), immunosuppressants (especially cyclosporin) · Localised weakness o infective - osteomyelitis o vascular - infarction o Neoplastic § primary - multiple myeloma (#1), osteosarcoma (#2 - bimodal age incidence distribution), chondrosarcoma (#3) § secondary - PB-KTL (prostate, breast, kidney, thyroid, lung) + haematological cancers

A 50yo male presents to ED following a few months of intermittent epigastric abdominal pain and the pain has become dramatically worse in the last few hours. On exam, he has tachycardia and is hypotensive with a temperature of 37.8. He also has rebound tenderness and guarding over the whole abdomen.

My provisional diagnosis is bacterial contamination of the peritoneum following rupture of a duodenal or peptic ulcer. This is on the basis of severe abdominal pain, nausea and fever as well as the characteristic signs of peritonitis. My differential diagnoses on the basis of his epigastric pain include: · GIT: GORD, proximal GI tumours, gastritis and acute pancreatitis · Vascular: Aortic dissection, mesenteric ischaemia · Other: biliary colic, pleuritic pain, pericarditis Exclude inferior myocardial infarction

A 15-year-old boy presents with a 12 hr history of scrotal and abdominal pain, following a Saturday morning rugby game. On examination you find a swollen and tender scrotum. He was afebrile but distressed.

My provisional diagnosis is testicular torsion. · epidemiology: bimodal incidence à neonatal period and pubertal · aetiology: 5-10% of cases of testicular torsion are induced by trauma. the rest of the time torsion is spontaneous. Other causes of acute testicular pain include · testicular o mechanical § torsion of the testicular or epididymal appendages § incarcerated +/- strangulated indirect inguinal hernia o traumatic § haematoma § testicular rupture o infective - epididymo-orchitis o vascular § idiopathic testicular infarction § HSP/IgA vasculitis o (unlikely, but red flag) Malignant - testicular cancer · Non-testicular o Nephrolithiasis (wouldn't examine as testicular torsion)

Patient presents with blood in his sputum, hoarseness, has been smoking for 15 years. On auscultation there was dullness to percussion on the left upper lobe. Decreased air entry on the left upper lobe. He had recent difficulty breathing and difficulty with phonation.

PDx Lung carcinoma DDx Infective - Bronchitis - Pneumonia - TB Neoplastic - Metastatic Ca from elsewhere Systemic - Coagulopathy - Autoimmune disease o Goodpasture's Syndrome o Wegener's Granulomatosis aka Granulomatosis with Polyangiitis Other - Pulmonary Oedema - Inhaled Foreign body

Patient presents with blood in his sputum, hoarseness, has been smoking for 15 years. On auscultation there was dullness to percussion on the left upper lobe. Decreased air entry on the left upper lobe. He had recent difficulty breathing and difficulty with phonation. What are your provisional and differential diagnoses?

PDx Lung carcinoma DDx Infective - Bronchitis - Pneumonia - TB Neoplastic - Metastatic Ca from elsewhere Systemic - Coagulopathy - Autoimmune disease o Goodpasture's Syndrome o Wegener's Granulomatosis aka Granulomatosis with Polyangiitis Other - Pulmonary Oedema - Inhaled Foreign body

A 52-year-old patient presents with drooping eyelids, blurred vision and bitemporal hemianopia. There was a diagram of the visual field quadrants - you're asked to name this abnormality. Provisional and mechanism for blurred vision, doploipa and bitemporal hemianopia

Pituitary adenoma The pituitary gland sits in the sella turcica, a midline depression of the sphenoid bone where it is in close apposition to apposition (and can therefore compress) the · optic chiasm à when it enlarges it is at risk of compressing this structure. Classically, it initially affects the decussating optic nerve fibres from the temporal visual fields à bitemporal hemianopia · left and right cavernous sinuses -> eye palsies (CNIII, IV, VI), & loss of sensation on the face in the distribution of CNV1 and 2. NB: compresses medial structures first (CNVI, not ICA - too much pressure)

52-year-old patient Presented with headache and loss of sensation to forehead and side of nose and horizontal diplopia. Later developed ptosis, fixed and dilated pupil (Left eye), deviated inferiorly and laterally. Provisional diagnosis

Pituitary adenoma invading the cavernous sinus

A 52-year-old woman was noted to have a large mass in the right fornix during a routine pelvic examination. She had been in good health and specific questioning regarding vaginal bleeding or abdominal symptoms was negative. There had been no change in her bowel habit and no weight loss or anorexia. The involuted uterus was anteverted, and the cervix was mobile. What are the possible lesions that may result in a mass in the right fornix?

Preamble: Because of the way they are suspended within the pelvis by their respective ligaments (mesovarium & mesosalpinx portions of the broad ligament), the most likely anatomical location of a mass palpated in either of the lateral fornices is the ovaries or fallopian tube. Differentials · Ovarian & Tubal o Cystic § Benign cysts § Follicular - Graafian follicle doesn't rupture/release egg but continues to grow (associated with hyperestrogenism and endometrial hyperplasia) § corpus luteum - enlarged fluid-filled corpus luteum after failed regression, NB: continues to produce progesterone § theca-lutein - multiple cysts due to exaggerated simulation of theca interna cells secondary to excessive gonadotropins like B-hCG (associated with GTD) § Polycystic ovary § Endometriomas o Neoplastic § Benign tumours - cystadenomas (serous or mucinous), Brenner (some solid and/or calcified components) § Malignant § Primary § Surface epithelial § Germ cell § Sex cord stromal § Secondary ("Krukenberg" tumour) § Reproductive: § endometrial, uterine leiosarcoma § breast § Lymphoma § GIT - colorectal, gastric o Abscess · Other o Uterine pathology - congenital malformations, leiomyoma (fibroid), neoplasms, Lymphadenopathy

A woman presents to your clinic after missing her last menstrual period and subsequently taking a pregnancy test, which was positive 3 months ago. Over the last few days, she developed pelvic discomfort, small amounts of per vaginal bleeding, hyperemesis gravidarum and a distended uterus. On testing, her ß-HCG was double the normal range for a normal pregnancy. Ultrasound was performed and no foetal heart sounds or parts were identified.

Provisional diagnosis · GTD - Molar pregnancy given the positive urinary pregnancy screen, elevated ß-HCG and lack of identifiable foetal parts on US Differential diagnosis · Gestational states o Incorrect dates o Multiples o Ectopic pregnancy o Miscarriage (<20 weeks) o sentinel bleed for APH e.g., placental abruption · Malignancies o GTN - invasive molar pregnancy, Choriocarcinoma o Ovarian teratoma (B-hCG)

A 53-year-old postmenopausal women noticed some vaginal bleeding, left iliac fossa and medial thigh pain as well as urinary symptoms. Examination reveals mild ascites and on bimanual palpation you felt a pelvic mass and adductor weakness on left side.

Provisional diagnosis is ovarian cancer given the: · Palpable adnexal mass associated with ascites and urinary symptoms · Older age / post-menopausal Differential diagnoses: · Differentials for iliac fossa mass +/- post-menopausal bleeding: o Gynaecological § Neoplastic: cervical cancer, endometrial cancer, metastases § Benign: Meig's syndrome, ovarian cyst, endometriosis/adenomyosis, uterine polyps, fibroid (leiomyoma), vaginal atrophy § Infectious: STI, pelvic inflammatory disease/tubo-ovarian abscess § *Ectopic pregnancy in women of reproductive age o Non-gynaecological § Lymphatic: lymphoma, lymphadenopathy § Urinary: pyelonephritis, cystitis, transplanted kidney § GIT: rectal cancer § Haem: coagulopathy • Post-menopausal bleeding • Vaginal atrophy • Endometrial cancer • Uterine cancer • Cervical cancer • Metastases to the ovaries • Meigs' syndrome • Triad of ascites, hydrothorax and a benign ovarian tumour • Other • UTI • Dermoid cyst • Theca lutein cyst • Tubovarian abscess Mnemonic for abnormal PV bleeding - Not ALL relevant for postmenopausal bleeding • PALM-COEIN (PALM for structural pathology, COEIN for systemic pathology) • Polyps • Adenomyosis • Leiomyoma • Malignancy • Coagulopathy • Ovarian dysfunction • Endometrial • Iatrogenic • Not Yet Classified

Case: A 55-year-old man is brought by ambulance to the ED. He is complaining of nausea, vomiting and abdominal pain. His abdomen is soft, non-distended with a lump that "does not go away'' in the inguinal region. He has guarding to upper and lower abdomen with palpation. He has a history of diabetes. · Temp: 35.9 · HR: 100bpm · RR: 28 · BP: 157/80 · SpO2: 100% RA · BSL: 10.4 mmol/L What are your provisional and differential diagnoses?

Provisional diagnosis: SBO secondary to an inguinal hernia Note: Normally won't vomit for hernia, but only when it gets obstructed/strangulated/incarcerated Differential diagnoses · Mass above the inguinal ligament o inguinal hernia o lymphadenopathy o males (young) § undescended testis § spermatic cord lipoma or hydrocoele o females § cyst of the canal of Nuck · >> portion of the processus vaginalis within the inguinal canal in women · Mass below the inguinal ligament o femoral hernia o lymphadenopathy o saphena varix o femoral aneurysm · Main causes of small bowel obstruction 1. adhesions 2. hernias 3. small bowel tumours § Benign (95%) - adenoma, lipomas, fibromas, leiomyomas § Malignant (5%) · Primary: #1 carcinoid, #2 adenocarcinomas, lymphomas, GIST · Secondary: e.g., melanoma 4. strictures (e.g., IBD) · Red flag causes of abdominal pain (differentials for SBO) o GI § Ischaemic colitis § Ascending cholangitis § PUD § Pancreatitis § Appendicitis o Reproductive § Ectopic pregnancy (female) § Torsioned testicle or ovary

A 56yo man presents with a 5-week history of polyuria, and increased thirst as well as feeling lethargic. He has a background history of hypertension and hypercholesterolemia. On examination, he is an obese gentleman with dry mucous membranes. Vitals are as follows: BP 140/95, HR 84, T 37.1

Provisional diagnosis: T2DM Differentials: - Late onset type 1 diabetes - Pancreatic insufficiency o Haemochromatosis o Chronic pancreatitis o Pancreatic carcinoma - Drug induced o Glucocorticoid excess o Lithium induced polydipsia o Thiazides and other diuretics - Endocrine o Diabetes insipidus à deficiency of vasopressin/ADH - Nephrogenic causes à resistance to ADH - Acromegaly -> excess GH/IGF-1 o Cushing's syndrome à leading to glucose intolerance o o Hyperthyroidism

A 28 year old sexually active female presents with dysuria, frequency and discharge. Results of MSU: WBC <10^6, lots of epithelial cells, 2x bacteria both <10^8/L. 2 bacteria included E coli and enterococcus faecalis.

Provisional diagnosis: UTI Differentials I would consider include · Renal o Pyelonephritis o Urethritis § Mechanical inflammation of the urethra secondary to passage of a kidney stone o Transitional cell carcinoma (unlikely given age and gender) o Glomerulonephritis à haematuria · Gynaecological o Infections § STI/PID (upper reproductive tract): · bacterial: gonorrhoea, chlamydia, mycoplasma genitalium, trichomonas vaginalis § Vaginitis (lower reproductive tract, not sexually transmitted) - bacterial vaginosis, candida · Others o Diabetes mellitus à frequency, polyuria o Medications eg. diuretics à frequency

74-year-old man presented with loin pain 28-hours post-op TURP and is haemodynamically unstable. On urinalysis, blood+, protein-, white cells-, nitrites-. He is taking anti-hypertensives, NSAIDs, diuretics and ARB.

Provisional diagnosis: Urosepsis: septic shock likely secondary to pyelonephritis given his haemodynamic instability (hypotensive, tachycardic) and urinary symptoms Differentials for acute loin pain · Renal pathology o Obstructive uropathy § nephrolithiasis (e.g., staghorn calculi) § malignancy - RCC, TCC, invasive prostate or cervical cancer § BPH (not relevant in this man) · Retroperitoneal pathology o AAA · Other o GI pathology § RUQ - Hepatitis § Epigastric - Ascending cholangitis, Pancreatitis, PUD § RIF - Appendicitis o Reproductive pathology § Males: testicular torsion, epididymo-orchitis § Females: ovarian torsion or complication (rupture, haemorrhage), ectopic pregnancy Differential for haemodynamic instability · Haemorrhagic hypovolemic shock (post-operative complication) · septic shock originating from alternative source, e.g. o lower lobe pneumonia (RUQ/flank pain) o (females) severe PID

A 16-year-old patient presents with anorexia, nausea and abdominal pain that started in the mid-abdomen and later localised to the right lower quadrant. What is your provisional and differential diagnosis?

Provisional diagnosis: appendicitis GIT(likely distal small bowel given pattern of pain) - Infectious o Mesenteric adenitis o Gastroenteritis (likely distal small bowel given pattern of pain) - Obstruction o Inguinal hernia - Inflammation o Inflammatory bowel disease (Chron's) o Meckel's diverticulum (infants/toddlers) o Colonic diverticulitis (elderly) § Would usually have PR bleed, right sided is uncommon Urinary - Infectious o UTI o Cystitis o Pyelonephritis - Renal calculi Gynaecological (female) - Obstetric: o Ectopic pregnancy - Ovarian o Torsion o Cyst rupture o Cancer - Endometriosis - Infectious - PID Testicular (male) - Mechanical o Torsion - Infectious o Epidiymo-orchitis (inflammation of the epididymis and/or testicle)

A 37-year-old female presents with 12 days of low grade fever, cough productive of a little sputum, malaise and lethargy. Crackles over all lung fields on auscultation. She was seen by her GP, given amoxicillin and sent home. · HR 120 · BP 140/90 · RR 26 · Temp 37.8

Provisional diagnosis: community acquired pneumonia. Given this patients presentation, I suspect they are infected with an atypical organism, the most common being a mycoplasma pneumoniae. Differentials · Respiratory o infective § bacterial - typical and atypical, TB § viral (incl. COVID19, Influenza) § fungal (immune compromised) o non-infective § restrictive - ILD (Multisystemic inflammatory condition - e.g., SLE, RA) § vascular - PE § neoplastic - malignancy (primary or secondary) · Cardiac: o (Right heart) IE o Pulmonary oedema NB: differentials for acute cough · Foreign body aspiration · Inhalational of bronchial irritant - smoke or fumes · Asthma/COPD exacerbation · URTI, LRTI

Case: A 35-year-old female with a history of prior chlamydial infection presents with low-grade fever and non-specific lower abdominal pain. Examination reveals mild diffuse lower abdominal tenderness on deep palpation. She has cervical motion tenderness and a mucopurulent vaginal discharge on pelvic examination.

Provisional diagnosis: pelvic inflammatory disease Differential diagnoses: · Obstetric/Life-threatening: o ectopic pregnancy · Other infections o Lower reproductive tract infection (would explain discharge) § Cervicitis § Vulvovaginitis · Bacterial vaginosis: thin, white, fishy discharge + mild vulva irritation · Trichomoniasis · Thrush o Non-gynaecological infections § UTI § GI - appendicitis, diverticulitis · Non-infectious causes o Neoplasia of the reproductive tract (Vagina, cervix, endometrium, ovary) o Other causes of pelvic/abdominal pain (NB: many women may not present with fever and discharge) § endometriosis

A 52-year-old tradesman who drives a van is currently on carbamezapine 400mg BD for epilepsy. He's suffered from 2 seizures in the past 6 months. He takes his medication erratically and has recently gotten diplopia one hour after his morning dose. Serum levels reveal that it is the upper level of the normal range.

Provisional diagnosis: the most common cause of seizure in a patient with known epilepsy is poor adherence with medication. I suspect the new onset of diplopia is a side effect of his medication Differential diagnosis · Causes of reduced seizure threshold in an epileptic o Poor sleep o Psychosocial stress o Systemic infection · Causes of provoked seizures o vascular/bleed - TIA/stroke, AVM o traumatic brain injury o infective - meningitis/encephalitis (e.g., limbic) o metabolic - hypoglycaemia, electrolyte derangements (hypo-), hepatic or renal failure, porphyria o toxic/iatrogenic - drug withdrawal (benzos, alcohol) o SOL - neoplastic o degenerative - dementia (incl. hippocampal sclerosis) § NB: onset is usually late stage and the predominant seizure type is focal non-motor · Seizure mimics o Hypoxia -convulsive syncope secondary to an MI, tachyarrhythmia o (sleep) REM sleep disorder § patients act out dreams due to lack of usual muscle atonia during this sleep stage o pseudoseizure

A 55-year-old man presents after a new onset seizure. His wife witnessed the episode, which involved twitching and weakness of the face that appeared to spread from the upper left limb. He recently had some terrible headaches. He also experienced sensory changes and decreased memory and his wife noted personality changes.

Provisional: Partial seizure secondary to a space-occupying lesion in the frontal and parietal lobes, likely a brain tumour or brain metastasis given his history of memory and personality changes Signs of space-occupying lesion-> simple focal seizures and headaches - Frontal lobe à decreased memory and personality changes -Precentral (frontal) and postcentral (parietal) gyri à sensory and motor changes Differentials for the episode o Atypical migraine o Stroke/TIA o Psychogenic Differentials for causes of a seizure: o Neoplastic o Benign tumour o Malignant tumour o Metastases o Infective o Abscess o Meningitis o Encephalitis o Vascular o Haemorrhagic stroke o Ischaemic stroke o TIA o Metabolic o Hypoglycaemia o Hypo/hypernatraemia o Epilepsy o Psychogenic non-epileptic seizure o Migraine with aura o Alcohol withdrawal

A 61-year-old woman presented to the Emergency Department complaining of abdominal pain. The pain was of gradual onset over the previous four hours and was now severe and confined to the epigastrium. On examination there was epigastric tenderness with guarding and rigidity. Bowel sounds were absent, and there was no evidence of hepatosplenomegaly or lymphadenopathy

Provisional: "Given this patients presentation with gradual onset of severe epigastric pain, guarding and rigidity and absent bowel sounds, my provisional diagnosis is acute pancreatitis complicated by peritonitis and distributive shock. This is a medical emergency. NB: peritonitis occurs early in pancreatitis because the organ has direct contact with the posterior parietal peritoneum. paralytic ileus develops due to inflammation. Importantly this patient is tachypnoeic, tachycardic and hypotensive and is therefore demonstrating signs of shock." Differentials for epigastric abdominal pain: - GI o Pancreas: pancreatic cancer o Stomach: perforated PUD, gastritis o Gallbladder: acute cholecystitis, ascending cholangitis o Liver: acute hepatitis o Small and large bowels: IBD, coeliac disease o Kidneys: pyelonephritis, renal calculi - Cardiovascular: o inferior MI, ruptured AAA, aortic dissection and hypovolaemic shock - Respiratory o basal pneumonia and septic shock - Spleen o Infarction Radiation to back à retroperitoneal - Aortic o Aneurysm impending rupture o Dissection (would be acute onset) - Renal o Pyelonephritis o Nephrolithiasis - Life-threatening thoracic pathology o Cardiorespiratory § Inferior MI § Pulmonary embolism

A 27-year-old male presents to ED following an injury during a football game. He twisted his right knee and heard a pop. On examination, the patient could weight bear but the knee felt unstable and it was tender and swollen. Lachman's test, the anterior drawer and pivot shift were positive.

Provisional: ACL injury Differentials for acutely swollen knee (somewhat mechanism dependent) · Alternative or concomitant ligamentous tear (MCL, LCL, PCL (NB: requires significant force) and associated inflammation · Meniscal injury (NB: less swelling) · Bone contusion or intra-articular fracture (unlikely given the usual mechanism of injury for an ACL) · Patella pathology - subluxation/dislocation, tendon tear, fracture · Less common: quadriceps tendon tear, fibular head or neck fracture, tibiofemoral dislocation

A 56-year-old woman with a history of T2DM noticed increasing pain and swelling in her right calf after a hip replacement surgery. Examination revealed an obese woman with distinctive swelling of the right calf. Vitals showed hypertension and tachypnoea.

Provisional: DVT Differentials: Unilateral swelling o Exudate - non-pitting oedema § Cellulitis § Septic arthritis § lymphoedema o Transudate - pitting oedema §Venous insufficiency/ compression from tumour, abscess or other mass -> pelvic mass compressing venous structures § NOT RHF, cirrhosis, nephrotic syndrome -> would cause bilateral swelling · Mass o calf muscle haematoma Leg pain o muscular § compartment syndrome (a painful condition caused by too much pressure in the muscles eg. from fracture (eg. tibia), exercise, crash injury) § myositis (many causes à likely to be bilateral) Skeletal § osteomyelitis § (traumatic mechanism) fracture Ruptured baker's cyst -> popliteal fossa tenderness

A 35-year-old man presented with progressive wasting, malaise and severe dysphagia for solids and liquids. He had unprotected sex with male partners over the previous 15 years. Physical examination showed a wasted appearance and a fever of 38.2 ̊C. White plaques were on the tonsils, palate and tongue.

Provisional: HIV infection Differentials:[RM1] · Sore throat and generalised lymphadenopathy o Infectious (URTI) § Viral - EBV, COVID-19, adenovirus, rhinovirus § Bacterial - GAS · Generalised rash and lymphadenopathy o Infectious § Viral exanthem - · measles, rubella, roseola (HH6), parvovirus B19 · Ross River, Barmah Forest § Bacterial · STI's: HIV, secondary syphilis · Streptococcal infections o meningococcal o Scarlet fever (strep. Pyogenes) · Rickettsia infections e.g., Rocky Mountain Spotted Fever (tick-borne) o Adverse drug reaction o Inflammatory § Kawasaki (can occur in adults) · Lymphadenopathy o Infectious § viral - CMV § bacterial - mycobacteria, brucellosis, coxiella burnetti § parasitic - toxoplasmosis o Autoimmune/infiltrative - sarcoidosis, amyloidosis, SLE o Neoplastic (could explain cachexia and fever) § Primary - lymphoma § Secondary (cervical) - leukaemia, head/neck, lung, breast, [RM1]HIV Or Other infections including - those that might explain the patient's sore throat (URTI - viral: EBV, bacterial: GAS) - those that might explain the patient's generalised rash (viral exanthem, bacterial: STI's - secondary syphilis, meningococcal) An adverse drug reaction (could explain lymphadenopathy & rash) Inflammatory conditions including Kawasaki (more common in children), SLE Neoplastic causes including lymphoma

A 35-year-old man was seen in consultation because of progressive wasting, malaise and severe odynophagia for solids and liquids. He reported having unprotected sex with several male partners but had previously avoided screening for HIV. Physical examination revealed a wasted appearance, mild dehydration and a fever of 38.2°C. There were white plaques on the tonsils, palate and tongue. The remainder of the physical examination was normal. Investigations confirmed that he was HIV-positive. His CD4 count was 0.1 x 109/L (reference range 0.5-1.4 x 109/L). A full blood count was also performed (see opposite).

Provisional: HIV infection Differentials:[RM1] · Sore throat and generalised lymphadenopathy o Infectious (URTI) § Viral - EBV, COVID-19, adenovirus, rhinovirus § Bacterial - GAS · Generalised rash and lymphadenopathy o Infectious § Viral exanthem - · measles, rubella, roseola (HH6), parvovirus B19 · Ross River, Barmah Forest § Bacterial · STI's: HIV, secondary syphilis · Streptococcal infections o meningococcal o Scarlet fever (strep. Pyogenes) · Rickettsia infections e.g., Rocky Mountain Spotted Fever (tick-borne) o Adverse drug reaction o Inflammatory § Kawasaki (can occur in adults) · Lymphadenopathy o Infectious § viral - CMV § bacterial - mycobacteria, brucellosis, coxiella burnetti § parasitic - toxoplasmosis o Autoimmune/infiltrative - sarcoidosis, amyloidosis, SLE o Neoplastic (could explain cachexia and fever) § Primary - lymphoma § Secondary (cervical) - leukaemia, head/neck, lung, breast, [RM1]HIV Or Other infections including - those that might explain the patient's sore throat (URTI - viral: EBV, bacterial: GAS) - those that might explain the patient's generalised rash (viral exanthem, bacterial: STI's - secondary syphilis, meningococcal) An adverse drug reaction (could explain lymphadenopathy & rash) Inflammatory conditions including Kawasaki (more common in children), SLE Neoplastic causes including lymphoma

A man presented with a 3-week history of bilateral cervical lymphadenopathy, a sore throat, malaise and a generalised rash. He is a previous IVDU user.

Provisional: HIV infection Differentials:[RM1] · Sore throat and generalised lymphadenopathy o Infectious (URTI) § Viral - EBV, COVID-19, adenovirus, rhinovirus § Bacterial - GAS · Generalised rash and lymphadenopathy o Infectious § Viral exanthem - · measles, rubella, roseola (HH6), parvovirus B19 · Ross River, Barmah Forest § Bacterial · STI's: HIV, secondary syphilis · Streptococcal infections o meningococcal o Scarlet fever (strep. Pyogenes) · Rickettsia infections e.g., Rocky Mountain Spotted Fever (tick-borne) o Adverse drug reaction o Inflammatory § Kawasaki (can occur in adults) · Lymphadenopathy o Infectious § viral - CMV § bacterial - mycobacteria, brucellosis, coxiella burnetti § parasitic - toxoplasmosis o Autoimmune/infiltrative - sarcoidosis, amyloidosis, SLE o Neoplastic (could explain cachexia and fever) § Primary - lymphoma § Secondary (cervical) - leukaemia, head/neck, lung, breast, [RM1]HIV Or Other infections including - those that might explain the patient's sore throat (URTI - viral: EBV, bacterial: GAS) - those that might explain the patient's generalised rash (viral exanthem, bacterial: STI's - secondary syphilis, meningococcal) An adverse drug reaction (could explain lymphadenopathy & rash) Inflammatory conditions including Kawasaki (more common in children), SLE Neoplastic causes including lymphoma

A 32-year-old single mother with 2 children under the age 5 comes to see you, her GP, complaining about having headaches and lack of energy. She is unemployed and divorced 6 months ago. On further questioning, she reveals that she has had little appetite and has lost 5 kg in the last two months. She struggles to fall asleep, often wakes up in the middle of night and finds it hard to get back to sleep. She has lost interest in doing things that she used to enjoy, like walking and gardening, as well as worrying about her parenting. She finds herself losing her temper easily and yells at her children, leading her to believe that she is a bad mother. She often feels like crying for no apparent reason. On discussing the situation with this woman, you believe that she is suffering from major depression. You discuss options for therapy, including CBT and pharmacotherapy. She does not feel she can commit to CBT at the present time.

Provisional: Major depressive disorder o At least one major depressive episodes. o Depressed mood and/or anhedonia for at least 2 weeks and at least 4 other symptoms that cause impairment in social, work areas of functioning Differentials: · Primary mood disorders o Adjustment disorder o Chronic situational depression o Bipolar disorder depressive episode o Anxiety · Organic depression (caused by injury or disease affecting brain tissues as well as by chemical or hormonal abnormalities) o Neurological vs Non-neurological § Neurological · Neurodegenerative disease -> Alzheimers, Parkinsons; · Space-occupying lesion · Malignancy · Collagen vascular disease (e.g. SLE) § Non-neurological · Endocrinology - hypothyroidism · Chronic renal failure · Anaemia · Other malignancy (especially lymphoma and pancreatic carcinoma or carcinoid syndrome) · Viral illness (e.g. mononucleosis) · Medications - steroids, beta blockers, alcohol

A 70yo lady, currently on hypertensive medications was brought in by her daughter who is worried because she has noticed her mother has been disinterested lately. She has lost weight, isn't sleeping well and can't seem to concentrate

Provisional: Major depressive disorder o At least one major depressive episodes. o Depressed mood and/or anhedonia for at least 2 weeks and at least 4 other symptoms that cause impairment in social, work areas of functioning Differentials: · Primary mood disorders o Adjustment disorder o Chronic situational depression o Bipolar disorder depressive episode o Anxiety · Organic depression (caused by injury or disease affecting brain tissues as well as by chemical or hormonal abnormalities) o Neurological vs Non-neurological § Neurological · Neurodegenerative disease -> Alzheimers, Parkinsons; · Space-occupying lesion · Malignancy · Collagen vascular disease (e.g. SLE) § Non-neurological · Endocrinology - hypothyroidism · Chronic renal failure · Anaemia · Other malignancy (especially lymphoma and pancreatic carcinoma or carcinoid syndrome) · Viral illness (e.g. mononucleosis) · Medications - steroids, beta blockers, alcohol

A 9-month old child presents with persistent cough resulting in vomiting. Her mother says she has been feeding poorly, been irritable and not been able to be settled. On examination, you hear an inspiratory 'whoop'. What are your provisional and differential diagnoses?

Provisional: Whooping cough AKA pertussis Differentials: · Infections o Other childhood infections: § Bronchiolitis (respiratory syncytial virus) § Croup (laryngotracheobronchitis - parainfluenza virus) o Other bacterial pneumonia · Inhaled foreign body · Anaphylaxis · Asthma exacerbation (in an older child)

A 45-year-old female school teacher presents to ED with a 3-week history of fever, especially in the afternoon and evening, associated with night sweats. Her appetite was poor, but she denied weight loss and has had mild abdominal discomfort. Physical examination revealed a temperature of 38.5°C, a regular HR of 88, and a BP of 130/80, and mild diffuse abdominal tenderness. Examination was otherwise unremarkable

Provisional: fever of unknown origin -> 4 criteria: · >3 week duration · Fever > 38.3ºC on at least 2 occasions · Inconclusive history, examination and initial investigations o No clear aetiology despite investigations on 3 outpatient visits, or 3 days in the hospital, or 1 week of invasive ambulatory investigation · Not immunocompromised PUO refers to a prolonged febrile illness without an establish aetiology despite intensive evaluation and diagnostic testing

A 23-year-old man fell on an outstretched arm during a rugby game. On examination his arms were asymmetrical, swollen and erythematous. His arm was externally rotated and the head of humerus was not palpable in the joint. There was also loss of sensation in the upper lateral arm.

Provisional: glenohumeral joint dislocation Given the mechanism of injury I would also like to rule out concomitant · Upper limb o fracture - (1/4 of dislocations involve clinically significant fractures) § Humeral head - Hill-Sachs, Bankart, avulsion # of the greater tubercle § Humeral shaft § Elbow (depending on mechanism) - Monteggia or Galleazzi § Clavicle o joint disruption § Acromioclavicular § Sternoclavicular · Lower limb o Fracture - colles

A 65-year-old female, who had a 10-year history of hypertension and a myocardial infarction three years previously, presented to the Emergency Department with sudden onset of weakness in the left arm and leg. On examination, her pulse was rapid and irregularly irregular. She had a left VII cranial nerve palsy of the upper motor neuron type and left spastic hemiparesis, with an ipsilateral upgoing plantar response

Provisional: haemorrhagic stroke in the posterior limb of the internal capsule... (secondary to longstanding HTN and anticoagulant therapy) àPosterior limb because pure motor stroke - Provisional: pure motor right-sided stroke o Sudden onset of focal neurological deficits o Site of the lesion and pathological processes o The clinical syndrome of pure unilateral motor weakness is most commonly due to disruption of the densely-packed motor fibres of the internal capsule (posterior limb) Differential diagnoses I would consider include · vascular - TIA, ischaemic stroke (thromboembolic, hyaline arteriolosclerosis), haemorrhagic bleed (intracerebral, SAH, subdural, epidural), vasculitis · infectious - meningitis, encephalitis, abscess, acute disseminated encephalomyelitis · metabolic o seizure with post-ictal Todd's paresis o nonketotic hyperosmolar hyperglycaemic stupor (can present with focal neurological deficits - motor and/or sensory deficits, seizures) · hemiplegic migraine · autoimmune - acute attack of MS · neoplastic - tumour · intoxication

A 65yo woman presents after a fall. She had recovered initially. She has a background of AF, HTN, and T2DM. Her medications include metformin, anti-hypertensives, and DOAC therapy. On exam, she had left lower limb weakness and loss of sensation. There was a left facial droop, as well as hyperreflexia and a positive Babinski reflex. HR 120, BP 230/110, T 37.6, RR 14.

Provisional: haemorrhagic stroke in the posterior limb of the internal capsule... (secondary to longstanding HTN and anticoagulant therapy) àPosterior limb because pure motor stroke - Provisional: pure motor right-sided stroke o Sudden onset of focal neurological deficits o Site of the lesion and pathological processes o The clinical syndrome of pure unilateral motor weakness is most commonly due to disruption of the densely-packed motor fibres of the internal capsule (posterior limb) Differential diagnoses I would consider include · vascular - TIA, ischaemic stroke (thromboembolic, hyaline arteriolosclerosis), haemorrhagic bleed (intracerebral, SAH, subdural, epidural), vasculitis · infectious - meningitis, encephalitis, abscess, acute disseminated encephalomyelitis · metabolic o seizure with post-ictal Todd's paresis o nonketotic hyperosmolar hyperglycaemic stupor (can present with focal neurological deficits - motor and/or sensory deficits, seizures) · hemiplegic migraine · autoimmune - acute attack of MS · neoplastic - tumour · intoxication

A 12-year-old Chinese girl had problems concentrating in class. She had symptoms of hyperthyroidism and blood results revealing high T3/T4, low TSH What is your provisional and differential diagnoses?

Provisional: hyperthyroidism, secondary to Graves disease Differentials · Causes of a hyperfunctioning thyroid gland o Graves - the #1 cause of hyperthyroidism (60-80%) o Toxic multinodular goitre (15-20%) o Toxic adenoma (5%) o Malignancy (all types = rare) § follicular thyroid carcinoma § TSH-producing pituitary adenoma · Causes of destruction of the thyroid gland causing release of hormone o Thyroiditis § Autoimmune - Hashitoxicosis § Infectious - De Quervains (subacute granulomatous) § Pregnancy related · Gestational · (pathological) B-hCG mediated (hydatidiform mole, choriocarcinoma) o B-hCG stimulates the TSH receptor · Post-partum (subacute lymphocytic AKA silent) o Iatrogenic - radiation thyroiditis · Ectopic (extrathyroidal) hormone production o Ovarian teratoma · Exogenous iodine or thyroid hormone o Excess iodine § Contrast § Amiodarone § Diet o Levothyroxine overdose

A 42-year-old woman was referred to the Endocrinology Clinic with suspected acromegaly. The patient had noticed gradually worsening bifrontal headache for the past six months. There were no visual disturbances. The patient had also noticed changes in her physical appearance especially enlargement of her hands and feet. There was a history of arthralgia in the knees, a feeling of increasing lethargy and tiredness, and of nocturia. Physical examination revealed a woman with coarse facial features, enlargement of her jaw and tongue and a small goitre. Her hands were large and spade-like, blood pressure was 140/100. There was cardiomegaly and evidence of median nerve compression in the right hand. Crepitus was present in both knees. There was proximal muscle weakness in the upper and lower limbs. The remainder of the neurological examination was normal. The ocular fundus revealed no evidence of papilloedema, and visual fields examination was normal.

Provisional: pituitary adenoma · headache: mass effect · blurry vision: failure of accommodation due to compression of the parasympathetic fibres of CNIII palsy +/- diplopia due to CNIII, CNIV or CNVI palsies · thirst: central DI à failure to concentrate the urine · weight gain, fatigue: hypothyroidism Differentials: · sellar: craniopharyngioma, Rathke's pouch cyst · parasellar: ICA aneurysm, meningioma

A 47-year-old post-menopausal woman presents with worsening headaches, blurry vision, increasing thirst, tiredness & weight gain of 10kg over 6-8 weeks.

Provisional: pituitary adenoma · headache: mass effect · blurry vision: failure of accommodation due to compression of the parasympathetic fibres of CNIII palsy +/- diplopia due to CNIII, CNIV or CNVI palsies · thirst: central DI à failure to concentrate the urine · weight gain, fatigue: hypothyroidism Differentials: · sellar: craniopharyngioma, Rathke's pouch cyst · parasellar: ICA aneurysm, meningioma

An 86-year-old lady presented with gradual vision loss on a background of asthma and family history of open angle glaucoma.

Provisional: primary open-angle glaucoma (peripheral vision loss and FMHx) Differentials: · other types of glaucoma: secondary open-angle, normal tension, chronic angle-closure · other causes of progressive vision loss o Acquired refractive errors - presbyopia § More congenital à myopia (defocused for distance), hyperopia (defocused for near), astigmatism, o Cataracts - progressive blurring, defocus, colour desaturation, glare at night o Macular degeneration - central field vision loss o Diabetic or hypertensive retinopathy - mid-peripheral blurring of vision, focal scotomas etc. o Papilloedema - increased ICP

Margaret is a 64-year-old lady who has had progressive worsening of her peripheral vision over the last 12 months. Her ophthalmologist suspected the diagnosis of open-angle glaucoma and arranged for some investigations. She has a history of asthma (on inhaled salbutamol) but is otherwise well

Provisional: primary open-angle glaucoma (peripheral vision loss and FMHx) Differentials: · other types of glaucoma: secondary open-angle, normal tension, chronic angle-closure · other causes of progressive vision loss o Acquired refractive errors - presbyopia § More congenital à myopia (defocused for distance), hyperopia (defocused for near), astigmatism, o Cataracts - progressive blurring, defocus, colour desaturation, glare at night o Macular degeneration - central field vision loss o Diabetic or hypertensive retinopathy - mid-peripheral blurring of vision, focal scotomas etc. o Papilloedema - increased ICP

A 36-year-old woman presents to ED with peripheral cyanosis and a silent chest on a background of asthma. Salbutamol was not effective. Her vitals were as follows: heart rate 150, blood pressure 90/50, respiratory rate 36 with normal urinalysis.

Provisional: shock secondary to status asthmasticus Differentials: - Anaphylaxis - Foreign body obstruction - Pneumothorax - PE - Pneumonia - Bronchitis

A 72-year-old woman presents with 3-week history of retrosternal pain radiating to her throat and difficulty swallowing solids and sometimes liquids. She particularly finds it difficult to keep food down. Examination reveals a mildly distressed woman. but with no obvious physical abnormalities.

Scleroderma involves increased collagen deposition and fibrosis → damage to small BV → intramural neuronal damage → smooth muscle atrophy (not striated so only lower 2/3) → loss of tone in lower ⅔ → loss of peristalsis and hypomotility + weakness/incompetence of LES Up to 90% patients with scleroderma have oesophageal dysphagia Scleroderma Either the localised version (CREST syndrome), or the systemic version (Systemic Scleroderma) Differential diagnoses: o Neuromuscular (difficulty with solids and liquids, can be intermittent) § Primary: achalasia (diffuse oesophageal spasm, defect of myenteric plexus), myasthenia gravis § Secondary: scleroderma, myotonic dystrophy, bulbar palsy, pharyngeal paralysis o Mechanical (normal lumen 2-3cm diameter, dysphagia if lumen <1.3cm) (solids but not liquid affected, always there) § Intraluminal: foreign body § Luminal: · Oesophogeal cancer adenocarcinoma · Strictures (progressive): GORD, medication-induced (NSAIDs), chemotherapy/radiation-induced · Pharyngeal pouch: neck bulge · Oesophageal webs: extension of normal mucosal tissue, usually anterior post-cricoid · Oesophageal rings (Schatzki rings): concentric smooth extension of oesophageal tissue § Extraluminal: · Enlarged aorta or left atrium · Mediastinal masses: lymphadenopathy, retrosternal goitre, bronchial tumour o Functional dysphagia - dysphagia in the absence of other major oesophageal motor disorders, no structural or mucosal abnormality accounting for the symptom and no GORD or eosinophilic oesophagitis.

A 65-year-old man presented to the Emergency Department with a six-hour history of interscapular pain, urinary retention and mild bilateral leg weakness. On further questioning, he revealed that he had felt unwell for six weeks and had lost 3 kg in the past month. On examination he had a distended bladder, signs of corticospinal dysfunction in both legs, decreased sensation to the level of his rib cage and reduction in vibration sense below his iliac crest.

Spinal cord compression secondary to prostate cancer metastasis Given this patient's history of acute onset urinary retention, bone pain and bilateral neurological symptoms, my provisional diagnosis is spinal cord compression. (NB: the bilateral nature of this patient's neurological deficits make a brain lesion less likely). Spinal cord compression is a medical emergency. Given his recent malaise and weight loss, the most likely cause in this clinical scenario is malignant spinal cord compression. Potential sources of metastatic malignancy include: · Most likely: lung, prostate, (females - breast) · Other solid cancers: CRC, renal cell carcinoma, thyroid follicular carcinoma, melanoma · Haematological cancers: lymphoma, multiple myeloma This patients age and gender (65yo M) make prostate cancer the most likely cause. Differentials · Other causes of spinal cord compression include o Neoplastic - primary bone cancer (less likely than secondary) § Benign - osteoblastoma § Malignant - osteosarcoma, chondrosarcoma o mechanical § pathological fracture - osteoporosis, Paget's disease § intervertebral disc prolapse (herniation of the nucleus pulposus through the fibres of the annulus fibrosis) · NB: likely to cause radicular pain rather than point tenderness, also more common in the lumbar spine (this patient's point tenderness is in the thoracic spine) o infective - discitis, pyogenic vertebral osteomyelitis (risks: immunocompromise, IVDU), tuberculosis (Pott's' disease) o vascular - spontaneous spinal epidural haematoma à anticoagulant therapy, coagulopathy o iatrogenic - epidural hematoma or abscess (might develop secondary to lumbar puncture/epidural block à not relevant in this case) · spinal stroke

A 24-year-old single man presented to his GP with a two-week history of lethargy, anorexia, nausea and arthralgia. Over the past two days he had noticed darkening of his urine. On examination he was found to have scleral icterus, an enlarged tender liver (16 cm liver span) and splenomegaly.

The causes of jaundice can be classified based on whether there are elevated levels of unconjugated (pre-hepatic or hepatic) or conjugated (post-hepatic) bilirubin in the blood. Because this patient has developed jaundice and dark stools associated with a large tender liver on a background of prodromal illness (lethargy, anorexia, nausea, arthralgia) my provisional diagnosis is hepatic jaundice likely in the context of viral hepatitis. Causes of hepatic jaundice include · infectious o viral: § hepaviridae (most likely B +/-D, C, because A & E present as acute gastrointestinal illness) § herpesviruses - EBV, (immunocompromised patients à CMV, HSV, VZV) § adenovirus` § yellow fever, dengue fever o bacterial: salmonella typhi (typhoid fever), Brucella, Coxiella, Leptospirosis o parasitic: amoebiasis, toxoplasmosis, schistosomiasis · metabolic - alcoholic, NASH · genetic (may develop clinical signs anytime between young adulthood & middle age) - haemochromatosis, Wilson's disease, a-1 anti-trypsin · immune - autoimmune hepatitis, o NB: not PBC or PSC which = obstructive jaundice = pale stools · toxins/drugs o prescription medications - LOTS, #1: paracetamol, also antibiotics, anti-convulsants o ingestion: mushrooms · neoplastic - o primary - hepatocellular carcinoma o secondary · (pregnancy) HELLP · vascular - o hypoperfusion - sepsis o thrombosis: Budd-Chiari syndrome

A 30-year-old IVDU presented with pleuritic chest pain, haemoptysis, fever and a new onset murmur at the left sternal edge, as well as retinal haemorrhages on fundoscopy

What are your provisional and differential diagnoses? Provisional diagnosis: Infective endocarditis Differentials for pleuritic chest pain +/- dyspnoea, haemoptysis, fever · Cardiac o Non-bacterial thrombotic endocarditis/Marantic endocarditis § Secondary to Malignancy, Libman Sacks endocarditis (SLE) o Acute rheumatic fever (haemoptysis due to pulmonary oedema) o myo-/pericarditis - e.g., viral or uraemic o atrial myxoma o myocardial infarction · Respiratory o Pneumonia o PE with infarction o Lung malignancy Other potentially life-threatening causes of chest pain · Cardiac o Myocardial infarction · Respiratory o pneumothorax

A 45-year-old man complained of progressive lethargy and swelling of the ankles over the past three weeks. Examination revealed a pale and ill-looking man with pitting oedema of both ankles. The JVP was raised 3 cm, there was a fourth heart sound and crepitations were heard at both lung bases. Microscopy of fresh urinary sediment showed granular and red cell casts. Over the next 24 hours the total urine volume was 400 ml.

This patient has an AKI. Furthermore, they are presenting with clinical and biochemical findings suggestive of nephritic syndrome (glomerular haematuria: dysmorphic RBCs and urine RBC casts, proteinuria, oedema, hypertension, oliguria, azotemia and elevated creatinine). DIFFERENTIALS FOR AKI · pre-renal (perfusion issue) a) shock à acute tubular necrosis § cardiogenic - massive MI, complete heart block, cardiac failure § hypovolemic - e.g. blood loss, vomiting, diarrhoea § distributive (systemic vasodilation) - e.g. sepsis, neurogenic shock b) vascular § large-medium sized vessels (infarction) - aortic dissection, systemic thromboembolus, renal v. thrombosis c) cardiorenal syndrome d) hepatorenal syndrome · renal a) vascular § small vessel - causes of microangiopathic haemolytic anaemia (e.g. DIC, TTP, HUS, malignant hypertension, eclampsia), vasculitis b) glomerular - nephritic > nephrotic syndromes c) tubular § medications: NSAIDs, ACEI, diuretics (MOI à ischaemia) § toxin - iodine contrast induced, pigment (secondary to haemolysis or rhabdomyolysis) d) interstitial § drugs (75%) - antibiotics, NSAIDs and 5-ASA, immunomodulators, PPIs, allopurinol § systemic rheumatological disease - e.g. SLE, sarcoidosis, sjogren's, § infections · post-renal (obstructions à can also think anatomically: urethral, cystic, ureteral) a) stones +/- strictures b) tumours (TCC - bladder, prostate cx, cervical cx) § BPH c) iatrogenic - e.g. blocked IDC, strictures

A 62yo man comes to clinic with visual problems and a terrible headache which began earlier in the day. He reports bumping into objects and not being able to read half the printed page of the newspaper. He has a history of HTN and diabetes. On exam, there is right homonymous hemianopia with right face and limb sensory loss. A CT brain non-contrast is performed. What is the lesion that caused the deficit, and what areas are likely to be affected?

This patient has likely suffered a haemorrhagic stroke in the region of the PCA. The homonymous hemianopia is explained by impaired perfusion of the occipital lobe à affecting left primary visual cortex. The facial and limb sensory loss can be explain by impaired perfusion of either the internal capsule or the thalamus because the penetrating choroidal arteries often arise from the PCA.

A 44-year-old female clerical worker was brought into the Emergency Department following a "grand mal" seizure of two minutes duration, witnessed by her partner. She had never had any fits before and was in good health apart from headaches that had been present for several months, which she had attributed to the stress of her job. In fact, her employer had dismissed her only one week before, on the grounds that she had become forgetful and slow. She was initially drowsy, with bilateral upgoing plantar responses, though these returned to normal after 4 hours. The remainder of the physical examination was normal.

This patient's history of chronic headache, cognitive decline and new onset seizure at 44 years of age makes me suspicious of an intracranial space-occupying lesion such as a tumour. My provisional diagnosis is chronic raised ICP, likely secondary to a neoplastic space-occupying lesion.. Causes of new onset adult seizure include · neoplastic o benign tumours § neurocutaneous syndromes - neurofibromatosis, tuberus sclerosis, VHL o malignant tumours § primary § (more common) secondary - lung, breast/prostate cancer, melanoma, CRC, pancreatic cancer, RCC § paraneoplastic syndromes · vascular/bleed - TIA/stroke, AVM · infective - meningitis/encephalitis (e.g., limbic) · inflammatory - MS · trauma · metabolic - hypoglycaemia, electrolyte derangements (hypo-), hepatic or renal failure · iatrogenic - drug withdrawal (benzos, alcohol) · degenerative - dementia (incl. hippocampal sclerosis) o NB: onset is usually late stage and the predominant seizure type is focal non-motor · hypoxic - convulsive syncope Causes of increased ICP include à loosely based on the Monro-Kellie doctrine (see below) · (brain) SOL o neoplastic o infective § abscess § cysts - tapeworm, toxoplasmosis § granulomas - TB o NB: very rarely demyelinating disease (e.g., tumefactive MS, ADEM) or vasculitides (e.g., SLE) can mimic tumours · (csf) hydrocephalus o communicating § increased CSF production (rare - choroid plexus papilloma) § decreased resorption due to the scarring or obliteration of the arachnoid villi à e.g., after bacterial meningitis or a stroke § idiopathic AKA benign intracranial HTN AKA pseudotumour cerebri (mismatch due to cause unknown) o non-communicating § obstructive · congenital à e.g., Chiari or Dandy-Walker malformations · acquired à brain tumours o (excessive CSF production) § choroid plexus papillomas o (prevention of CSF outflow) § Ependymomas § pinealomas (intraventricular foramen) § medulloblastomas, astrocytomas (4th ventricle) · (blood) increased blood volume o haematoma (SDH) o venous outflow obstruction (venous sinus thrombosis, SVC obstruction) · cerebral oedema o osmotic - hyponatremia, hepatic encephalopathy o vasogenic (vessel damage) - hypertensive crisis o cytotoxic (cell death) - hypoxia

Case: A 47-year-old woman was hurrying for a bus when she experienced a sudden violent headache and collapsed. She was assisted by a friend, who noted that the patient was conscious but confused and in pain. Examination in the Emergency Department revealed a thin female, disoriented in time and place, with prominent nuchal rigidity. The remainder of the examination was normal apart from a sub-hyaloid haemorrhage in the right optic fundus.

This patient's onset of sudden severe headache in the context of physical exertion (PQ: argument) and associated with signs of meningism (nuchal rigidity, PQ: photophobia) makes me suspicious of subarachnoid haemorrhage, most likely caused by a ruptured berry aneurysm. Differentials I'd like to consider include · other intracranial haemorrhages o intracerebral § hypertensive crisis § from an AVM § Into a tumour § coagulopathy o subdural, epidural (NB: both unlikely because of required mechanisms) · vascular causes/infarction o stroke - arterial or venous infarction o septic embolism o dissection o vasculitis o spasm o cocaine abuse · seizure · primary headache o cluster

Past Question: A 56yo man with a background of HTN and smoking presents with severe headache occurring suddenly after a heated argument with his wife. He now experiences photophobia and there is nuchal rigidity.

This patient's onset of sudden severe headache in the context of physical exertion (PQ: argument) and associated with signs of meningism (nuchal rigidity, PQ: photophobia) makes me suspicious of subarachnoid haemorrhage, most likely caused by a ruptured berry aneurysm. Differentials I'd like to consider include · other intracranial haemorrhages o intracerebral § hypertensive crisis § from an AVM § Into a tumour § coagulopathy o subdural, epidural (NB: both unlikely because of required mechanisms) · vascular causes/infarction o stroke - arterial or venous infarction o septic embolism o dissection o vasculitis o spasm o cocaine abuse · seizure · primary headache o cluster

A 35-year-old woman presented to ED unwell with fever following a recent dental procedure. On examination she was unwell and had splinter haemorrhages on nail beds

What are your provisional and differential diagnoses? Provisional diagnosis: Infective endocarditis Differentials for pleuritic chest pain +/- dyspnoea, haemoptysis, fever · Cardiac o Non-bacterial thrombotic endocarditis/Marantic endocarditis § Secondary to Malignancy, Libman Sacks endocarditis (SLE) o Acute rheumatic fever (haemoptysis due to pulmonary oedema) o myo-/pericarditis - e.g., viral or uraemic o atrial myxoma o myocardial infarction · Respiratory o Pneumonia o PE with infarction o Lung malignancy Other potentially life-threatening causes of chest pain · Cardiac o Myocardial infarction · Respiratory o pneumothorax

A 56-year-old man was seen in the Haematology Clinic for review. He was diagnosed with polycythaemia vera six years ago, and had been treated with regular venesections. Over the past few months he had noticed increasing tiredness, early satiety, some weight loss and "discomfort" in the left upper quadrant of his abdomen. Examination revealed a cachectic man with conjunctival pallor. The splenic edge was palpable 10 cm below the costal margin in the mid-clavicular line. The remainder of his examination was normal. What complication(s) of his polycythaemia may account for the changes in his condition?

This presentation is most likely explained by the development of myelofibrosis or a leukemic transformation (in particular CML) of his PCV. Given that he has recently developed symptoms of anaemia (tiredness) and has massive splenomegaly, this suggests transformation to myelofibrosis. Other possibilities include CML and AML (less likely considering significant splenomegaly) - Myelofibrosis (MF). Is a rare type of blood cancer affecting the bone marrow and the production of bone of blood cells, and is characterized by the buildup of scar tissue within the bone marrow - Splenomegaly is due to sequestration of RBCs and extramedullary haematopoiesis

62F presents with palpitations and dizziness. OE: obese, HR 108 irregularly irregular. Given the following ECG strip: (irregularly irregular narrow-complex rhythm with absent p waves)

This rhythm strip shows an irregularly irregular narrow-complex rhythm with absent p waves à atrial fibrillation = a supraventricular tachyarrhythmia characterised by uncoordinated atrial activation (300-600bpm) resulting in an irregular ventricular response due to intermittent conduction through the AV node. Aetiology: There are reversible causes of AF à · sympathomimetic - o phaeochromocytoma o excess caffeine, cocaine and/or amphetamines · electrolyte imbalances - hypomagnesemia, hypokalaemia · endocrine - thyrotoxicosis · infection - fever · drugs - acute alcohol consumption Ultimately however, AF usually develops in the context of one or more risk factors, only 15% of patients with AF present with no apparent risk factors ("idiopathic" AF). These risk factors include: · General vascular risk factors - older age, HTN, smoking, DM, obesity, OSA · Intrinsic cardiac o IHD o VHD (esp. mitral stenosis à causes dilation of the RA & lengthening of conduction pathways), o cardiomyopathies o CHF o arrhythmogenic § pre-excitation tachycardia (e.g. WPW), § sick sinus, § congenital channelopathies, o infective - pericarditis · Non-cardiac o Pulmonary disease (COPD, PE, pneumonia etc.) o CKD o Chronic excessive alcohol consumption

A 69 year old male patient presents with haematuria and nocturia.

Transitional cell carcinoma

A 39-year-old woman presents with a 3-day history of flank pain and dysuria. On examination, she is febrile at 39 degrees, her blood pressure is 90/60mmHg and her heart rate is 120 bpm.

Urosepsis

Case: A 35-year-old homosexual male presents with right upper quadrant pain with 2 week history of nausea, vomiting, diarrhoea. In his early 20s, he has a history of IVDU but has since claimed to stop. While overseas in South Africa, he got a few tattoos on his arms and legs. On examination, he has hepatosplenomegaly and jaundice with some protein in the urine (GN). LFTs: · elevated AST/ALT · normal GGT/ALP, · normal bilirubin, normal total protein, albumin, normal globulin

What are your provisional and differential diagnoses? Provisional diagnosis: Acute Viral Hepatitis with obstructive jaundice due to hepatotropic viruses (viruses that have a special affinity for the liver) (exclude fulminant hepatitis) Differential Dx · Other Infective o Viruses: CMV, EBV, HSV, VZV, Adenovirus o Bacteria: Q-fever, Syphilis, Yellow fever · Metabolic o Wilson's Disease o α-1 anti-trypsin deficiency (excessive abnormal A1AT proteins in the liver) o Hemochromatosis · Drugs o Alcoholic hepatitis o Drug induced hepatitis: PAISA - Paracetamol, Aspirin, Isoniazid, Sulphonamide, Amiodarone · Neoplastic o Hepatocellular Carcinoma o Metastases to liver o Leukaemia/Lymphoma · Post-Hepatic (biliary tree obstruction) o Choledocolithiasis o Pancreatic cancer o Cholangiocarcinoma o Primary biliary cirrhosis o Primary sclerosing cholangitis · Other o Autoimmune hepatitis o NASH o Budd Chiari

A 32-year-old woman presented to her GP with a two-month history of stiffness in her knuckles and fingers. She also felt tired. Examination revealed swelling and tenderness of MCP 2-4 and PIP 1-5 bilaterally. There were small effusions in her knees. Over the counter ibuprofen has not assisted. Her vital signs are summarised in the table below.

What are your provisional and differential diagnoses? • Provisional o The provisional diagnosis is rheumatoid arthritis. The clinical picture of RA is of malaise, tiredness and stiffness in affected joints most prominent in the morning. The distribution is symmetrical and commonly affects MCPs and PIPs of the hands, MTPs and PIPs of the feet. Other joints commonly affected include mid tarsal, ankles, knees, hips, wrists, shoulders, upper neck. Affected joints are swollen and tender. • Inflammatory causes: o Spondyloarthropathies (seronegative) - ankylosing spondylitis, psoriatic arthritis, reactive arthritis, IBD arthritis o Connective tissue disease (seropositive) - SLE, Sjogren's syndrome, polymyositis/dermatomyositis, scleroderma o Crystalline (less likely as multiple joints and symmetrical) - gout, pseudogout o Post viral inflammatory causes such as Ross River and EBC o Bacterial causes such as chlamydia and gonococcus • Non-inflammatory causes (less likely given age + distribution of joints affected) - trauma, OA

A 26-year-old man has injured his left chest and abdomen in a car accident. He experiences severe chest pain and shortness of breath. Given a CT chest.

What is your diagnosis? Given the mechanism of injury and the patient's presentation with acute severe chest pain and dyspnoea, my provisional diagnosis is a tension pneumothorax. Differentials Differentials I'd like to consider include · Respiratory o pulmonary contusion (blood and oedema in the alveoli) o Haemothorax (could cause hypovolemic shock) o Chylothorax (secondary to thoracic duct injury) · MSK o Flail chest · Cardiac o Haemopericardium -> cardiac tamponade § due to direct trauma to the heart, great vessels and pericardial vessels à (Beck's triad) o Aortic dissection · GIT o oesophageal perforation (transmural: Boerhaave's) o Traumatic pancreatitis (seat belt) I'd like to consider the possibility that an acute medical event caused the accident (presenting due to car crash) · acute cardiac event - MI, dysrhythmia · respiratory - PE

A 20-year-old man was brought in by ambulance to the emergency department after a motor vehicle accident with complaints of right-sided chest pain and shortness of breath. He states that these symptoms began suddenly. He has no significant past medical history but has smoked cigarettes since the age of 16 years. The following are his vital signs: - Heart rate (b/min) 140 - Blood pressure (mmHg) 90/65 - Respiratory rate (/min) 24 - Temperature (0 C) 36.8

What is your diagnosis? Given the mechanism of injury and the patient's presentation with acute severe chest pain and dyspnoea, my provisional diagnosis is a tension pneumothorax. Differentials Differentials I'd like to consider include · Respiratory o pulmonary contusion (blood and oedema in the alveoli) o Haemothorax (could cause hypovolemic shock) o Chylothorax (secondary to thoracic duct injury) · MSK o Flail chest · Cardiac o Haemopericardium à cardiac tamponade § due to direct trauma to the heart, great vessels and pericardial vessels à (Beck's triad) o Aortic dissection · GIT o oesophageal perforation (transmural: Boerhaave's) o Traumatic pancreatitis (seat belt) I'd like to consider the possibility that an acute medical event caused the accident (presenting due to car crash) · acute cardiac event - MI, dysrhythmia · respiratory - PE

Case: A 55-year-old female with a past history of breast cancer presents with haemoptysis and dyspnoea. She has a 3-month history of fever, fatigue and 6kg weight loss. She is a non-smoker. On respiratory examination, there is stony dullness to percussion and decreased breath sounds bilaterally

What is your diagnosis? Given this patient's medical history and clinical presentation with constitutional signs and symptoms of malignancy and dyspnoea and haemoptysis and stony dullness to percussion, my provisional diagnosis is a malignant pleural effusion secondary to pulmonary metastases from her breast cancer primary. NB: I would also like to assess vocal resonance to differentiate between a pleural effusion and consolidation. VR is clear in consolidation & atelectasis and muffled in pleural effusion and a pneumothorax The most common causes of a pleural effusion include CHF, pneumonia and malignancy. One way I like to remember important differentials is based on whether the aetiology is localised or systemic · localised disease o infection § respiratory à pneumonia § other: subphrenic abscess, hepatic or splenic abscesses o malignant effusion § primary · direct spread of bronchogenic carcinoma · pleural cancer - mesothelioma § secondary metastases o infarction (PE) o inflammatory/rheumatological disease - RA o chylothorax (secondary to trauma, cardiothoracic surgery, lymphoma) · systemic disease o fluid overload à increased hydrostatic pressure § Right heart failure § Renal failure (low urine output) o Hypoalbuminaemia à decreased oncotic pressure § Cirrhosis § Nephrotic syndrome o drug-induced

A 40-year-old woman presents with cough and purulent sputum, associated with shortness of breath. She is febrile, tachycardic and tachypnoeic. Examination reveals coarse crepitations in the right base.

What is your provisional and differential diagnoses? · Provisional diagnosis: community acquired pneumonia given the symptoms of productive purulent cough, associated with fever, tachycardia, dyspnoea, tachypnoea and coarse crepitations in the right base · Differential diagnosis: o Respiratory § Infective: · Hospital-acquired or aspiration pneumonia · Lung abscess/empyema / necrotizing pneumonia · TB · Acute exacerbation of COPD/asthma · Acute bronchitis · Bronchiectasis: chronic cough associated with significant mucous production · Pharyngitis § Malignancy: Lung cancer (eg. bronchogenic carcinoma with overlying infection) § Vascular: PE: unlikely but life-threatening § Genetic: CF § Cardiac · Acute pulmonary oedema from sudden cardiac failure · PE · ACS § Interstitial · Pulmonary fibrosis · Infiltrative lung disease - sarcoidosis, asbestosis, silicosis

A 36-year-old woman presented to her local doctor after noticing a lump in her breast. She had neck pain and weakness in her left upper limb. She had weak elbow flexion. Loss of pain was also present from her right index finger to the ulnar aspect of her wrist.

· My provisional diagnosis is a breast neoplasm, which has metastasized cervical spine C5/6 o Lesion location:: anterolateral spinal cord compression -Spinal metastases at the level of C5-C6 on the LEFT side causing: § Ipsilateral C5-C6 LMN motor signs (elbow flexion = C5/6) § Contralateral C8 pain and temperature loss Differentials for breast lump: Benign neoplastic causes Fibroadenoma Malignant neoplastic causes Invasive ductal carcinoma (80%): fibrosis and stroma as a response to cancer, i.e. desmoplasia, hence feels rock hard. If it travels along the duct to the skin it becomes Paget's disease Invasive lobular carcinoma (5%) Medullary carcinoma: type of invasive ductal carcinoma but the tumour is soft, like the medulla in the brain Colloid carcinoma: type of invasive ductal carcinoma, tumour cells 'float' in pools of mucin Tubular carcinoma: best breast cancer to have, also a type of invasive ductal carcinoma, well differentiated tubular structures, string of beads pattern Other invasive cancers Non-neoplastic Breast cyst: due to fluid accumulation within breast glands Fibrocystic change: usually presents as bilateral chest pain that relates to menstrual cycle, worse in the luteal phase Non proliferative fibrocystic change: due to oestrogen and progesterone, very normal, occurs in 60-70% of women. Pathology: Fibrosis, dilated ducts/cysts, metaplasia of cells lining the ducts Proliferative fibrocystic change (ductal hyperplasia): Due to hormones, polyclonal proliferation of myoepithelial + secretory cells >2 cell layers and irregular fenestra (leave irregular spaces in duct). If cells are atypical this can increase the risk of breast cancer Basically it's in the name Fibro (fibrosis) Cystic (cysts i.e. dilated ducts) Change that is either Proliferative (proliferation), or Non-proliferative (metaplasia) Abscess/infection: rare, most likely mastitis from breast feeding Differentials for focal spinal lesion Vertebral osteomyelitis TB depositing in her vertebrae Herniated disc Vertebral fracture compressing the exiting spinal roots Spondylolisthesis Discitis

A 36-year-old woman presented to her local doctor after noticing a lump in her breast. On examination, there was 1cm mass in the lower right quadrant of the right breast that was firm and non-tender without attachment to deeper structures. On neurological examination, she has weakness of her right upper limb including weak elbow/wrist extension and loss of triceps reflex. Loss of pain and sensation was also present from the right side below the axilla.

· My provisional diagnosis is a breast neoplasm, which has metastasized to right side of C7 cervical spine. o The weak right wrist extension suggests lesion at C6-C7 right side of spine o The weak right elbow extension suggests a lesion at C7-8 right side of spine o Loss of sensation from T1 is consistent with a C7 lesion on the right impacting the decussated spinothalamic tract. Differentials for breast lump: Benign neoplastic causes Fibroadenoma Malignant neoplastic causes Invasive ductal carcinoma (80%): fibrosis and stroma as a response to cancer, i.e. desmoplasia, hence feels rock hard. If it travels along the duct to the skin it becomes Paget's disease Invasive lobular carcinoma (5%) Medullary carcinoma: type of invasive ductal carcinoma but the tumour is soft, like the medulla in the brain Colloid carcinoma: type of invasive ductal carcinoma, tumour cells 'float' in pools of mucin Tubular carcinoma: best breast cancer to have, also a type of invasive ductal carcinoma, well differentiated tubular structures, string of beads pattern Other invasive cancers Non-neoplastic Breast cyst: due to fluid accumulation within breast glands Fibrocystic change: usually presents as bilateral chest pain that relates to menstrual cycle, worse in the luteal phase Non proliferative fibrocystic change: due to oestrogen and progesterone, very normal, occurs in 60-70% of women. Pathology: Fibrosis, dilated ducts/cysts, metaplasia of cells lining the ducts Proliferative fibrocystic change (ductal hyperplasia): Due to hormones, polyclonal proliferation of myoepithelial + secretory cells >2 cell layers and irregular fenestra (leave irregular spaces in duct). If cells are atypical this can increase the risk of breast cancer Basically it's in the name Fibro (fibrosis) Cystic (cysts i.e. dilated ducts) Change that is either Proliferative (proliferation), or Non-proliferative (metaplasia) Abscess/infection: rare, most likely mastitis from breast feeding Differentials for focal spinal lesion Vertebral osteomyelitis TB depositing in her vertebrae Herniated disc Vertebral fracture compressing the exiting spinal roots Spondylolisthesis Discitis

An 11-year-old female patient comes to see you as a new patient at your general practice. You notice that she presents with reduced stature, webbed neck and a shield-shaped chest. Further investigations reveal renal malformation, diminished IQ and aortic coarctation.

· PDx: Turner's Syndrome (most commonly 45XO) · DDx: · Noonan Syndrome (very similar phenotype but not due to chromosomal abnormality, can therefore affect males) · Down's Syndrome (T21) · Klinefelter's (47XXY) · Edwards Syndrome (T18)

A 42-year-old male presents to ED severely dehydrated with fluctuating consciousness. According to his wife, he currently has the flu. She noticed that he had been thirsty and urinating more frequently. He has a BMI of 32 and his BSL are elevated.

· Provisional Dx: hyperosmolar hyperglycaemic state, a complication of diabetes · Differential Dx: o DKA with late onset as a complication of T1DM: unlikely in this patient as it more associated with Type 1 diabetes, so the patient would be younger, leaner with ketonuria o Other causes of severe dehydration: § Diabetes insipidus o Ketoacidosis § Alcoholic ketoacidosis § Starvation ketoacidosis: history of extremely little/nil intake. Inadequate carbohydrate availability à lipolysis and ketosis to provide fuel for cells o Acidosis § Lactic acidosis: if doing heavy exercise § Uraemic acidosis: background of chronic renal failure o Drug-induced § Paracetamol overdose § Salicylate overdose o Ingestion of toxic substances § Ethanol, methanol, ethylene glycol

Gerard is a 36-year-old miner who works several months a year in the mines. He presents with nausea and vomiting to the ED. He also describes stools that do not "stick together". He has palmar erythema, spider naevi and is jaundiced with a mild fever. His history is remarkable for alcohol consumption of 12-15 beers a day when he is "on the job".

· Provisional diagnosis is alcoholic hepatitis with liver cirrhosis o The significant history of alcohol consumption o Hepatic impairment § Subsequent symptoms of decreased detoxification (increased oestrogen à palmar erythema, spider naevi, and jaundice) o Triad of non-specific symptoms: nausea + vomiting + mild fever is common in alcoholic liver disease · Differential diagnoses for hepatitis include: o Infective § Viral hepatitis A-E § EBV § CMV § Viral gastroenteritis o Autoimmune § SLE § Autoimmune hepatitis o Metabolic § Wilson's disease § Haemochromatosis § Alpha-1-antitrypsin deficiency o Drug-induced § Paracetamol § Aspirin § Isoniazid § Sulphonamide § Amiodarone o Neoplastic § HCC § Liver metastases

Case: A 26-year-old female presents with 6 weeks of bloody and mucous diarrhoea. She had a previous episode one year ago.

· Provisional diagnosis: IBD and likely ulcerative colitis given her age, female gender, prolonged bloody diarrhoea and history of recurrence · Differential differentials: o Autoimmune § Crohn's disease: rarely get bloody diarrhoea unless severe § Coeliac disease o Vascular § Ischaemic colitis o Infective § Gastroenteritis · Bacterial: Shigella, Salmonella, E. Coli, C. Jejuni, Yersinia · Viral: Norovirus, Rotavirus (not bloody) · Amebiasis: wouldn't get leukocytes in faeces as these organisms destroy WCCs o Inflammatory § Pseudomembranous colitis: C. diff from antibiotic use § Diverticulitis: left sided o Neoplastic § CRC: left sided o Psychiatric § Laxative abuse § Irritable bowel syndrome

Case: A 43-year-old female presents with two-week history of bloody diarrhoea, 12 times in last 10 hrs, pain in left and right upper quadrants, previous episode 2 years ago that resolved.

· Provisional diagnosis: IBD and likely ulcerative colitis given her age, female gender, prolonged bloody diarrhoea and history of recurrence · Differential differentials: o Autoimmune § Crohn's disease: rarely get bloody diarrhoea unless severe § Coeliac disease o Vascular § Ischaemic colitis o Infective § Gastroenteritis · Bacterial: Shigella, Salmonella, E. Coli, C. Jejuni, Yersinia · Viral: Norovirus, Rotavirus (not bloody) · Amebiasis: wouldn't get leukocytes in faeces as these organisms destroy WCCs o Inflammatory § Pseudomembranous colitis: C. diff from antibiotic use § Diverticulitis: left sided o Neoplastic § CRC: left sided o Psychiatric § Laxative abuse § Irritable bowel syndrome

Case: A 60-year-old man presented to his GP with right loin pain and macroscopic haematuria. He had been feeling unwell for six weeks, had lost 6 kg in weight and had felt feverish on several occasions. Examination revealed an obese, plethoric, elderly man who looked unwell

· Provisional diagnosis: Renal cell carcinoma given he has 2 out of the 3 for the classic triad (flank pain and haematuria and abdominal mass) · Differential diagnoses: pre, renal (CISTIC) and post o Pre-renal § Coagulopathy, bleeding disorders, anticoagulant use § Malignant hypertension § Infective endocarditis, vasculitis, renal artery stenosis + shock or MI → renal infarction § Rhabdomyolysis o Renal (CISTIC) § Cancer · RCC: Dull flank pain, longstanding fever and haematuria = classic triad of tubular epithelial cell RCC · Bladder cancer (transitional cell carcinoma): cell of origin is the urothelial cell. Hx of aniline dye (hairdresser) and smoking · Prostate cancer: obstructive Sx, bone pain, fatigue, weight loss § Infection · UTI/pyelonephritis: frequency, urgency, dysuria · IgA nephropathy glomerulonephritis: recent URTI/pharyngitis · Renal TB § Stones · Nephrolithiasis: calcium oxalate stones, previous episodes of intense flank pain, colicky, possible passed stones, dehydration, high salt intake, pain radiating into iliac fossa, nausea § Trauma · Kidney damage: direct injury · Urethra/bladder damage: catheter § Infarction/Inflammation · Embolisation to renal artery: listen for murmurs · Renal artery stenosis + shock or MI can rarely cause infraction § Congenital · Autosomal dominant polycystic kidney disease: family history, renal mass, haematuria, cyst infection, HTN, chronic renal failure, liver cysts, berry aneurysms, mitral valve prolapse o Post-renal § Infection: cystitis, prostatitis, urethritis § Ureteric and bladder calculi § Trauma: catheter § Malignancy: bladder cancer, prostate cancer, urethral cancer, BPH

A 63-year-old male presents with two-week history haemoptysis and a 2-month history of weight loss. He has smoked 2 packs of cigarettes a day for 45 years. Examination reveals tachypnoea, pleural effusion and a temperature OR a 40-year-old female, non-smoker, presents with fever, weight loss, lethargy. CXR shows mediastinal widening and lung collapse. Later they tell you they found adenocarcinoma at left bronchus with enlarged mediastinal lymph nodes What are your provisional and differential diagnoses?

· Provisional diagnosis: lung cancer, most likely squamous cell carcinoma in a male smoker, given the patient's age, prolonged history of haemoptysis and weight loss, as well as his significant smoking history · Differential diagnosis o Neoplastic § Another form of lung cancer · Small cell · Large-cell · Adenocarcinoma § Lung metastasis § Kaposi's sarcoma if immunocompromised o Infectious § Bronchitis § Bronchiectasis § Pulmonary TB § Pneumonia § Fungal infection § Lung abscess o Vascular § PE: however, would expect a more acute picture o Autoimmune § Good pastures syndrome: attacks collagen in air sacs of lungs § Wegener's granulomatosis: systemic vasculitis involving granulomatosis and polyangitis with haemoptysis due to pulmonary haemorrhage


Conjuntos de estudio relacionados

Final Quiz Health Data Content and Standards 1.1 (RHIA & RHIT)

View Set

CA TEST 1 - Chapters 1, 2, 5.1, 5.2

View Set

E10: Week 9 (avid, cajole, enhance, nuance, rudimentary)

View Set

STAT242: Principal Component Analysis

View Set

Newborn Nursing Care and Assessment NCLEX Questions

View Set

INFS2608 Lecture 7 - Transaction Management

View Set

Principles of Marketing Practice

View Set

Chapter 22: Nursing Management: Patients With Oral and Esophageal Disorders and Patients Receiving Gastrointestinal Intubation, Enteral, and Parenteral Nutrition

View Set

Chapter 23: New Deal America, 1929-1939

View Set